Neurology Flashcards

1
Q

Dose of rtPA

A

0.9mg/kg IV

max 90mg

How well did you know this?
1
Not at all
2
3
4
5
Perfectly
2
Q

Indications for rtPA

A
How well did you know this?
1
Not at all
2
3
4
5
Perfectly
3
Q

Contraindications for rtPA

A
How well did you know this?
1
Not at all
2
3
4
5
Perfectly
4
Q

ABCD2 score for TIA that requires admission

A

> 3

How well did you know this?
1
Not at all
2
3
4
5
Perfectly
5
Q

most common site for a hypertensive hemorrhage?

A

Putamen

other common sites: thalamus, cerebellum, pons

How well did you know this?
1
Not at all
2
3
4
5
Perfectly
6
Q

What is the cut-off diameter for the intracranial bleed that will warrant surgical intervention?

A

> 3cm

How well did you know this?
1
Not at all
2
3
4
5
Perfectly
7
Q

most common cause of sporadic acute encephalitis in immunocompetent adults?

A

Herpes virus

How well did you know this?
1
Not at all
2
3
4
5
Perfectly
8
Q

What malignancy is the most common cause of brain metastases?

A

lung

But MELANOMAS have the greatest propensity to
metastasize to the brain

How well did you know this?
1
Not at all
2
3
4
5
Perfectly
9
Q

All patients with unexplained new onset seizures should undergo ____

A

brain imaging eg MRI or CT to search for an underlying structural abnormality

How well did you know this?
1
Not at all
2
3
4
5
Perfectly
10
Q

First line medications for generalized onset tonic clonic

A

Lamotrigine and Valproic acid

How well did you know this?
1
Not at all
2
3
4
5
Perfectly
11
Q

First line medications for focal seizures

A

Lamotrigine
Carbamazepine
Oxcarbazepine
Phenytoin
Levetiracetam

How well did you know this?
1
Not at all
2
3
4
5
Perfectly
12
Q

First line medications for typical absence

A

Valproic acid
Ethosuximide
Lamotrigine

How well did you know this?
1
Not at all
2
3
4
5
Perfectly
13
Q

First line medications for atypical absence and myoclonic,atonic

A

Valproic acid
Lamotrigine
Topiramate

How well did you know this?
1
Not at all
2
3
4
5
Perfectly
14
Q

Type of dementia suggested by early presence of visual hallucinations and tendency to have delirium

A

Dementia with Lewy bodies

How well did you know this?
1
Not at all
2
3
4
5
Perfectly
15
Q

Most common cause of dementia

A

Alzheimer’s dmentia

How well did you know this?
1
Not at all
2
3
4
5
Perfectly
16
Q

Rapidly progressive dementia with myoclonus

A

CJD

How well did you know this?
1
Not at all
2
3
4
5
Perfectly
17
Q

Characteristic imaging of Alzheimer’s disease

A

Entorhinal cortex and hippocampal atrophy

How well did you know this?
1
Not at all
2
3
4
5
Perfectly
18
Q

Characteristic imaging of CJD

A

Cortical ribboning, and basal gannglia or thalamus hyperintensity on DWI/FLAIR/MRi

How well did you know this?
1
Not at all
2
3
4
5
Perfectly
19
Q

First clinical manifestation of Alzheimer’s disease

A

memory impairment

How well did you know this?
1
Not at all
2
3
4
5
Perfectly
20
Q

Histopath findings in Alzheimer’s disease

A

Neuritic plaques composed of AB amyloid neurofibrillary tangles composed of phosphorylated tau

How well did you know this?
1
Not at all
2
3
4
5
Perfectly
21
Q

Acetylcholinesterase inhibitors that may be used for Alzheimer’s disease

A

Donepezil, Rivastigmine, Galantamine

How well did you know this?
1
Not at all
2
3
4
5
Perfectly
22
Q

NMDA inhibitor that may be used for Alzheimer’s disease

A

Memantine

How well did you know this?
1
Not at all
2
3
4
5
Perfectly
23
Q

Most common FTD syndrome

A

Behavioral variant

social and emotional systems dysfunction manifests as apathy, disinhibitrion, compulsivity, loss of empathy and oveeating

How well did you know this?
1
Not at all
2
3
4
5
Perfectly
24
Q

Cardinal features of Parkinson’s disease

A

Bradykinesia
Rigidity
Resting Tremor
Postural instability

How well did you know this?
1
Not at all
2
3
4
5
Perfectly
25
Q

Most common mutations associated with PD

A

Glucocerebrosidase

How well did you know this?
1
Not at all
2
3
4
5
Perfectly
26
Q

Why is levodopa routinely combined with decarboxylase inhibitor for PD?

A

prevents peripheral metabolism to dopamine and the development of nausea/vomiting

Levodopa remains the most effective symptomatic tx for PD

How well did you know this?
1
Not at all
2
3
4
5
Perfectly
27
Q

What do you call the decreasing duration of benefit of Levodopa ?

A

weaing off effect

How well did you know this?
1
Not at all
2
3
4
5
Perfectly
28
Q

Deep brain timulation of the _____ or ____ has largely replaced ablation surgery

A

Subthalamic nucleus or Globus pallidus interna

How well did you know this?
1
Not at all
2
3
4
5
Perfectly
29
Q

In which cases should patients undergo a neuroimaging study prior to LP?

A

Head trauma
Immunocompromised
Malignancies
Focal neurologic findings

In contrast, if bacterial meningitis is suspected, give antibiotics PRIOR to neuroimaging and LP

How well did you know this?
1
Not at all
2
3
4
5
Perfectly
30
Q

Most common pathogen causing acute bacterial meningitis in immunocompetent adults

A

S. pneumoniae

How well did you know this?
1
Not at all
2
3
4
5
Perfectly
31
Q

Pathogens that are implicated in acute bacterial meningitis following neurosurgical procedures especially shunting

A

S. aureus
CONS

How well did you know this?
1
Not at all
2
3
4
5
Perfectly
32
Q

Classic triad of meningitis

A

Fever
Headache
Nuchal rigidity

How well did you know this?
1
Not at all
2
3
4
5
Perfectly
33
Q

Empiric therapy for acute bacterial meningitis for chilcren >3 months and adults < 55

A

Cefotaxime/ Ceftriaxone/Cefepime + Vanco

How well did you know this?
1
Not at all
2
3
4
5
Perfectly
34
Q

Empiric therapy for acute bacterial meningitis for adults > 55 and adults of any age with alcoholism or debilitating illnesses

A

Ampicillin + cefotaxime/Ceftri/Cefepime + Vancomycin

How well did you know this?
1
Not at all
2
3
4
5
Perfectly
35
Q

Empiric therapy for acute bacterial meningitis for hospital acquired meningitis, posttraumatic or postneurosurgery meningitis, neutropenic patients or patients with impaired cell mediated imunity

A

Ampicillin + ceftazidime/meropenem + vanco

so dapat may coverage for P. aeuruginosa

How well did you know this?
1
Not at all
2
3
4
5
Perfectly
36
Q

Empiric treatment for L. monocytogenes

A

Ampi + Genta

How well did you know this?
1
Not at all
2
3
4
5
Perfectly
37
Q

Treatment course for the following pathogens for acute bacterial meningitis
Meningococcus
Pneumococcus
Gram neg
L. monocytogenes

A

Meningococcus - 7 days
Pneumococcus - 14 days
Gram neg - 21 days
L. monocytogenes -21 days

How well did you know this?
1
Not at all
2
3
4
5
Perfectly
38
Q

Adjunctive therapy with dexamethasone improves outcome from bacterial meningitis particularly if due to this pathogen

A

Pneumococcal- most striking benefit

May also be given in Hib and N. meningitidis

Should be given 20 mins prior to abx
No benefit if given >6 hrs after

Dexa may decrease penetration of VAN in the CSF.
As a result, to assure reliable penetration of vancomycin into the CSF, children and adults are treated with vancomycin in a dose of 45–60 mg/kg per day. Alternatively, vancomycin can be administered by the intraventricular route.

How well did you know this?
1
Not at all
2
3
4
5
Perfectly
39
Q

Prophylaxis for close contacts of acute meningitis

A

Rifampin 600 mg q12
- not for pregnant women

Alternative: Azithromycin x1 dose or
IM ceftriaxone (250 mg) x 1 dose

How well did you know this?
1
Not at all
2
3
4
5
Perfectly
40
Q

Procedure of choice that is rapid , sensitive, specific identification of viral meningitis

A

CSF PCR

Highest sensitivity if taken within 48 hrs of symptom onset
More sensi that CSF viral culture which in general has poor sensitivity

How well did you know this?
1
Not at all
2
3
4
5
Perfectly
41
Q

Treatment for meningitis due to HSV, EBV and VZV

A

IV acyclovir followed by an oral drug for 7-14d

How well did you know this?
1
Not at all
2
3
4
5
Perfectly
42
Q

Most common etiologic agent for Viral meningitis

A

Enteroviruses

How well did you know this?
1
Not at all
2
3
4
5
Perfectly
43
Q

Causative agent that should be considered when focal findings are present and when involvement of the inferomedial frontotemporal regions of the brain is likely

A

HSV

How well did you know this?
1
Not at all
2
3
4
5
Perfectly
44
Q

Neuroimaging procedure of choice for viral encephalitis

A

MRI

How well did you know this?
1
Not at all
2
3
4
5
Perfectly
45
Q

Treatment for CMV encephalitis

A

Ganciclovi/ Foscarnet
Cidofovir

How well did you know this?
1
Not at all
2
3
4
5
Perfectly
46
Q

Classic triad of brain abscess

A

Headache
Fever
Focal neurologic deficit

How well did you know this?
1
Not at all
2
3
4
5
Perfectly
47
Q

Empiric tx for brain abscess in immunocompetent patients

A

Third or 4th gen + Metronidazole

How well did you know this?
1
Not at all
2
3
4
5
Perfectly
48
Q

Most common cause of chronic meningitis

A

TB

How well did you know this?
1
Not at all
2
3
4
5
Perfectly
49
Q

Myasthenia gravis is due to autoantibodies against

A

Acetylcholine receptors

however levels do NOT correlate with severity

How well did you know this?
1
Not at all
2
3
4
5
Perfectly
50
Q

Organ that must me checked when patient is diagnosed to have MG

A

Thymus

Thymus is abnormal in 75% of px (65% hyperplasia, 10% thymoma)

Chest CT/MRI may be used

How well did you know this?
1
Not at all
2
3
4
5
Perfectly
51
Q

Lambert Eaton is due to autoantibodies against

A

presynaptic calcium channels

How well did you know this?
1
Not at all
2
3
4
5
Perfectly
52
Q

Autoantibodies that are present in 40% of AcH receptor antibody negative patients with generalized MG

A

Muscle specific kinase

How well did you know this?
1
Not at all
2
3
4
5
Perfectly
53
Q

Treatment of choice for MG

A

Anticholinesterase drug pyridostigmine (Mestinon)

How well did you know this?
1
Not at all
2
3
4
5
Perfectly
54
Q

Muscarinic side effects of pyridostigmine may be treated by

A

Atropine/ Diphenoxylate/ Loperamide

How well did you know this?
1
Not at all
2
3
4
5
Perfectly
55
Q

Mainstay of chronic immunosuppresive tx for MG

A

Glucocorticoids

How well did you know this?
1
Not at all
2
3
4
5
Perfectly
56
Q

Most common cause of hereditary neuropathy

A

Charcot Marie Tooth disease

How well did you know this?
1
Not at all
2
3
4
5
Perfectly
57
Q

Neuropathic disorders that may be considered if with symmetric proximal and distal weakness with sensory loss

A

GBS and CIDP

How well did you know this?
1
Not at all
2
3
4
5
Perfectly
58
Q

First line tx for painful sensory neuropathies

A

Lidoderm
TCA (amitriptyline, nortriptyline)
Gabapentin, Pregabalin
Duloxetine

How well did you know this?
1
Not at all
2
3
4
5
Perfectly
59
Q

in GBS, maximum weakness is usually reached within ____ week/s

A

2

How well did you know this?
1
Not at all
2
3
4
5
Perfectly
60
Q

Variant of GBS that presents with ophthalmopareis, facial diplegia, ataxia, areflexia

A

Fisher syndrome

Assoc with antibody to GQ1b

How well did you know this?
1
Not at all
2
3
4
5
Perfectly
61
Q

Most common neuropathies (what nerve/s)

A

Ulnar/Median nerve in the arm
Peroneal/Fibular in the leg

How well did you know this?
1
Not at all
2
3
4
5
Perfectly
62
Q

Involved nerve in Saturday night palsy

A

Radial nerve

Presents with wrist drop

How well did you know this?
1
Not at all
2
3
4
5
Perfectly
63
Q

What can be given to px with Duchenne’s muscular dystrophy to slow progression of disease

A

Glucocorticoids may slow progression of dse for up to 3 yrs

How well did you know this?
1
Not at all
2
3
4
5
Perfectly
64
Q

Type of dystrophy with scapular winging

A

Fascioscapularhumeral dystrophy

How well did you know this?
1
Not at all
2
3
4
5
Perfectly
65
Q

Window period (in hours) for giving thrombolytics for stroke

A

May be extended up to 4.5 hours

How well did you know this?
1
Not at all
2
3
4
5
Perfectly
66
Q

Head injury that shows lenticular shaped collection the frontal convexity

A

Epidural

l-E-mon shaped = Epidural

How well did you know this?
1
Not at all
2
3
4
5
Perfectly
67
Q

Vessels involved in epidural hematoma

A

middle meningeal arteries

How well did you know this?
1
Not at all
2
3
4
5
Perfectly
68
Q

Vessels involved in subdural hematoma

A

bridging veins

Water under the bridge

Under= sub
Bridge = bridging veins

How well did you know this?
1
Not at all
2
3
4
5
Perfectly
69
Q

Triad of Wernicke’s disease

A

Ophthalmoplegia, ataxia and confusion

How well did you know this?
1
Not at all
2
3
4
5
Perfectly
70
Q

Duration of tonic clonic movements that would differentiate syncope vs seizure

A
How well did you know this?
1
Not at all
2
3
4
5
Perfectly
71
Q

Classic EEG finding in absence seizure

A

generalized symmetric 3hz spike and slow wave pattern

How well did you know this?
1
Not at all
2
3
4
5
Perfectly
72
Q

What antiseizure medication is associated with formation of renal stones

A

Topiramate

since it is a weak carbonic anhydrase inhibitor

Also assoc with glaucoma

How well did you know this?
1
Not at all
2
3
4
5
Perfectly
73
Q

The only drug that prevents paroxysmal hemicrania

A

Indomethacin

How well did you know this?
1
Not at all
2
3
4
5
Perfectly
74
Q

Type of headache that may be aborted by oxygen supplementation

A

Cluster

How well did you know this?
1
Not at all
2
3
4
5
Perfectly
75
Q

Prophylactic tx for cluster headache

A

Verapamil, Topiramate, Melatonin, Lithium

How well did you know this?
1
Not at all
2
3
4
5
Perfectly
76
Q

Abortive tx for SUNCT/SUNA

A

Lidocaine

How well did you know this?
1
Not at all
2
3
4
5
Perfectly
77
Q

Prophylactic tx for SUNCT/SUNA

A

Lamotrigine
Topiramate
Gabapentin

Lamotrigine –> most effective

How well did you know this?
1
Not at all
2
3
4
5
Perfectly
78
Q

Only proven treatment for chronic tension type headache is

A

TCA eg. amitriptyline

How well did you know this?
1
Not at all
2
3
4
5
Perfectly
79
Q

Most common pattern of nerve injury in HIV neuropathy

A

distal symmetric polyneuropathy

How well did you know this?
1
Not at all
2
3
4
5
Perfectly
80
Q

Gold standard for examining CSF in px suspected to have neurosyphilis

A

VDRL

How well did you know this?
1
Not at all
2
3
4
5
Perfectly
81
Q

Most common cause of SAH

A

Trauma

If outside trauma, ruptured saccular aneurysm

How well did you know this?
1
Not at all
2
3
4
5
Perfectly
82
Q

Focal seizures that spread to involve both cerebral hemispheres to produce generalized seizures usually originate from which lobe of the brain?

A

Frontal

How well did you know this?
1
Not at all
2
3
4
5
Perfectly
83
Q

What is the most common cause of
seizures in the older adults?

A

Cerebrovascular dse

How well did you know this?
1
Not at all
2
3
4
5
Perfectly
84
Q

The triad of ipsilateral tongue weakness, weakness of contralateral upper and lower extremities, and contralateral loss of vibration and proprioception is characteristic of lesions involving which structure?

A

medial medulla

Medial medullary syndrome (Dejerine Syndrome): clinical triad of ipsilateral hypoglossal palsy, contralateral hemiparesis, and contralateral lemniscal sensory loss (loss of vibration and proprioception)

Lateral medullary syndrome (Wallenberg syndrome): ipsilateral loss of facial pain and temp sensation, contralateral hemisensory loss of pain and temp sensation, ipsilateral losses (ataxia of arm/leg, gait, nystagmus, hoarseness, dysphagia, horner syndrome)

How well did you know this?
1
Not at all
2
3
4
5
Perfectly
85
Q

What are the degenerative proteins in the ff dementia:
Alzheimers
FTD
Lewy body
Huntington dse
CJD

A

Alzheimers - AB/Tau
FTD -Tau
Lewy body- alpha synuclein
Huntington dse - polyglutamine repeats
CJD - prions

How well did you know this?
1
Not at all
2
3
4
5
Perfectly
86
Q

Although seen only in a minority of patients, how does the classic headache associated with a brain tumor present?

A

Predominates in the morning and improves during the day

How well did you know this?
1
Not at all
2
3
4
5
Perfectly
87
Q

Therapy with dexamethasone for acute meningitis is unlikely to be of significant benefit if it started > hours after antimicrobial therapy is initiated.

A

6

Should be given 20 mins prior to abx

How well did you know this?
1
Not at all
2
3
4
5
Perfectly
88
Q

Hemiparesis is the most common localizing sign of an abscess located in which region of the brain?

A

frontal

How well did you know this?
1
Not at all
2
3
4
5
Perfectly
89
Q

Acute seizures occurring at the time of a stroke are most often seen in which type of strokes?

A

cardioembolic

How well did you know this?
1
Not at all
2
3
4
5
Perfectly
90
Q

Aside from brain imaging, what is the only other diagnostic test needed prior to treatment with IV rTPA?

A

CBG

How well did you know this?
1
Not at all
2
3
4
5
Perfectly
91
Q

Primary CNS lymphomas are commonly associated with this virus

A

EBV

How well did you know this?
1
Not at all
2
3
4
5
Perfectly
92
Q

Most common primary brain tumor

A

Meningioma

How well did you know this?
1
Not at all
2
3
4
5
Perfectly
93
Q

Most common mode of spread of tumors metastatic to the nervous system

A

Hematogenous

How well did you know this?
1
Not at all
2
3
4
5
Perfectly
94
Q

Primary rad onco approach to brain metastases

A

SRS
It can sterilze visible lesins and produce local disease control in 80-90% of px
Can tx up to 10 lesions gowever confined to lesions <= 3 cm and is most effective in <= 1 cm

previously whole brain RT

How well did you know this?
1
Not at all
2
3
4
5
Perfectly
95
Q

CSF glucose ____ and elevated CSF protein ___ are pedictive of increased mortality and poorer outcomes in px with meningitis

A

<40mg/dL
>3g/L

How well did you know this?
1
Not at all
2
3
4
5
Perfectly
96
Q

In acute viral meningitis PMNs may predominate in the first ___ hours

A

48 hours

How well did you know this?
1
Not at all
2
3
4
5
Perfectly
97
Q

Treatment for syphilitic meningitis

A

Aqueous Penicillin G 3-4 million units IV every 4 h for 10-14 days

How well did you know this?
1
Not at all
2
3
4
5
Perfectly
98
Q

Possible tx option for JCV

A

5-HT2a receptor antagonist mirtazapine

How well did you know this?
1
Not at all
2
3
4
5
Perfectly
99
Q

Stage of brain abscess formation wherein you can START to see ring enhancing capsules on imaging

A

Early capsule formation

If capsule + gliosis –> Late capsule formation; inc risk of seizure

How well did you know this?
1
Not at all
2
3
4
5
Perfectly
100
Q

Stage of brain abscess formation wherein pus starts to form

A

Late cerebritis stage

How well did you know this?
1
Not at all
2
3
4
5
Perfectly
101
Q

Minimum duration of tx for brain abscess

A

6-8 weeks

How well did you know this?
1
Not at all
2
3
4
5
Perfectly
102
Q

Most common type of primary headache

A

tension type

How well did you know this?
1
Not at all
2
3
4
5
Perfectly
103
Q

The release of this substance in the trigeminal nucleus triggers the underlying vascular mechanism of migraine headaches

A

Calcitonin gene related protein

How well did you know this?
1
Not at all
2
3
4
5
Perfectly
104
Q

The only primary headache more common in males

A

Cluster headache

How well did you know this?
1
Not at all
2
3
4
5
Perfectly
105
Q

Most common schwanommas

A

vestibular/ acoustic neuroma

NF2 have high incidence of vestibular schwanomma

How well did you know this?
1
Not at all
2
3
4
5
Perfectly
106
Q

Most common cause of hemorrhagic metastasis in the brain

A

Lung CA

but greatest propensity to cause hemorrhage: melanoma, thyroid, kidney CA (these are just not as common as lung CA)

How well did you know this?
1
Not at all
2
3
4
5
Perfectly
107
Q

Among patient who suffered from Acute Ischemic Stroke (AIS), a decrease in cerebral blood flow to zero causes death of brain tissue within

A

10 minutes

How well did you know this?
1
Not at all
2
3
4
5
Perfectly
108
Q

most common cause of cerebral embolism overall

A

Nonrheumatic atrial fibrillation

How well did you know this?
1
Not at all
2
3
4
5
Perfectly
109
Q

best sequence to detect brain infarction earlier than CT or other MR sequences.

A

Diffusion-weighted imaging (DWI)

How well did you know this?
1
Not at all
2
3
4
5
Perfectly
110
Q

This is a new drug which functions as a plasminogen activator that has the added advantage of bolus dosing without 1-hour infusion (vs alteplase). It can improve efficiency for patients that need to be transferred to another facility or for those who will undergo endovascular therapy

A

Tenecteplase

T for transfer

How well did you know this?
1
Not at all
2
3
4
5
Perfectly
111
Q

What constitutes ABCD2 score in TIA

A

Age- >= 60

BP (SBP >140 OR DBP >90)

Clinical symptoms (Unilateral weakness -2,
Speech disturbance without weakness -1)

Duration (>60 m -2, 10-59m -1)
Diabetes -1

How well did you know this?
1
Not at all
2
3
4
5
Perfectly
112
Q

in SAH Once ICP is recorded, CSF drainage (if available), osmotic therapy, and blood pressure management can be tailored to maintain cerebral perfusion pressure (MAP - ICP) at least ____ mmHg.

A

50–70

How well did you know this?
1
Not at all
2
3
4
5
Perfectly
113
Q

Most common seizure type resulting from metabolic derangements

A

Generalized Tonic clonic

How well did you know this?
1
Not at all
2
3
4
5
Perfectly
114
Q

EMG pattern that may help distinguish spasms from brief tonic and myoclonic seizures.

A

Characteristic rhomboid pattern

How well did you know this?
1
Not at all
2
3
4
5
Perfectly
115
Q

Most common syndrome associated with focal seizures with impairment of consciousness

A

Mesial Temporal Lobe Epilepsy Syndrome

Characteristic hippocampal sclerosis on MRI

How well did you know this?
1
Not at all
2
3
4
5
Perfectly
116
Q

Treatment for Mesial Temporal Lobe Epilepsy Syndrome

A

Refractory to treatment with anticonvulsants but responds well to surgical intervention

How well did you know this?
1
Not at all
2
3
4
5
Perfectly
117
Q

EEG finding in Mesial Temporal Lobe Epilepsy Syndrome

A

Unilateral or bilateral anterior temporal spikes

How well did you know this?
1
Not at all
2
3
4
5
Perfectly
118
Q

Example of antiseizure drugs with mechanism
Facilitating the opening of potassium channels

A

Ezogabine

How well did you know this?
1
Not at all
2
3
4
5
Perfectly
119
Q

Anti epileptic drug associated with gingiva hyperplasia

A

phenytoin

How well did you know this?
1
Not at all
2
3
4
5
Perfectly
120
Q

Anti epileptic drug associated with SJS

A

Lamotrigine

How well did you know this?
1
Not at all
2
3
4
5
Perfectly
121
Q

Initial tx of choice for status epilepticus

A

Benzodiazepine (Midazolam, Lorazepam, Diazepam)

How well did you know this?
1
Not at all
2
3
4
5
Perfectly
122
Q

In typical amnestic AD, brain atrophy begins in the

A

medial temporal lobes

How well did you know this?
1
Not at all
2
3
4
5
Perfectly
123
Q

most important risk factors for developing Alzheimer’s disease

A

age and family hx

How well did you know this?
1
Not at all
2
3
4
5
Perfectly
124
Q

What constitutes the triad of Huntington’s disease?

A

Chorea, behavioral disturbance, and executive impairment

How well did you know this?
1
Not at all
2
3
4
5
Perfectly
125
Q

Triad of Normal pressure hydrocephalus

A

clinical triad includes an abnormal gait (ataxic or apractic), dementia and urinary urgency or incontinence

How well did you know this?
1
Not at all
2
3
4
5
Perfectly
126
Q

Slow vertical saccades are seen in what type of dementia causing dse?

A

Progressive supranuclear palsy

How well did you know this?
1
Not at all
2
3
4
5
Perfectly
127
Q

What can be given to px with Lewy Body Dementia that is a selective inverse agonist of the serotonin 5-HT2A receptor that does not block dopamine receptors

A

Pimavanserin

Patients with Lewy Body Dementia should not be exposed to typical neuroleptics that can lead to a neuroleptic malignant syndrome and death, or anticholinergics or dopamine agonists that can exacerbate their symptoms.

How well did you know this?
1
Not at all
2
3
4
5
Perfectly
128
Q

one dopamine agonist is comparable to levodopa, but must be administered parenterally

A

Apomorphine

How well did you know this?
1
Not at all
2
3
4
5
Perfectly
129
Q

Most common parkinsonism?

A

Parkinson’s dse

How well did you know this?
1
Not at all
2
3
4
5
Perfectly
130
Q

Only oral agent for PD that has been to shown to reduce dyskinesia without worsening parkinsonian feature

A

Amantadine

How well did you know this?
1
Not at all
2
3
4
5
Perfectly
131
Q

What does F.A.S.T. stand for?

A

Facial weakness, Arm weakness, Speech abnormality, Time

How well did you know this?
1
Not at all
2
3
4
5
Perfectly
132
Q

Illegal drugs that have strong association with stroke

A

Cocaine and amphetamine

How well did you know this?
1
Not at all
2
3
4
5
Perfectly
133
Q

Expected EEG findings in HSV encephalitis

A

periodic, stereotyped, sharp-and-slow complexes

How well did you know this?
1
Not at all
2
3
4
5
Perfectly
134
Q

Progressive multifocal leukoencephalopathy is associated with what virus

A

JC virus

How well did you know this?
1
Not at all
2
3
4
5
Perfectly
135
Q

Size of brain abscess that may be treated medically

A

2-3 cm

How well did you know this?
1
Not at all
2
3
4
5
Perfectly
136
Q

Cushing reflex triad

A

Hypertension
Bradycardia
irregular breathing

How well did you know this?
1
Not at all
2
3
4
5
Perfectly
137
Q

CSF abnormalitis in bacterial meningitis

A

Note gluc <40; protein >45

How well did you know this?
1
Not at all
2
3
4
5
Perfectly
138
Q

Treatment for meningtis due to H. capsulatum

A

AmB (0-7-1.0 mg/kg/day) for 4-12 weeks (total of 30mg/kg is recommended)

How well did you know this?
1
Not at all
2
3
4
5
Perfectly
139
Q

Duration of chronic meningitis

A

> 4 weeks

How well did you know this?
1
Not at all
2
3
4
5
Perfectly
140
Q

Etiology of chronic meningitis associated with multiple cyst in basal meninges with hydrocephalus

A

cysticercosis

How well did you know this?
1
Not at all
2
3
4
5
Perfectly
141
Q

Treatment for SSPE

A

No definitive therapy for SSPE is available.

Treatment with isoprinosine (Inosiplex, 100 mg/kg per day), alone or in combination with intrathecal or intraventricular interferon-α, has been reported to prolong survival and produce clinical improvement in some patients but has never been subjected to a controlled clinical trial.

How well did you know this?
1
Not at all
2
3
4
5
Perfectly
142
Q

Stage of brain abscess formation wherein central core of coagulative necrosis begins to form

A

Early cerebritis

How well did you know this?
1
Not at all
2
3
4
5
Perfectly
143
Q

Stage of brain abscess formation wherein there is a well-formed necrotic center and dense collagenous capsule

A

Late capsule formation
Associated with gliosis which inc risk for seizure

How well did you know this?
1
Not at all
2
3
4
5
Perfectly
144
Q

in TIA The risk of stroke after a TIA is ~___% in the first__ months, with most events occurring in the first __ days.

A

10–15
3
2

How well did you know this?
1
Not at all
2
3
4
5
Perfectly
145
Q

Target BP to reduce risk of stroke

A

< 130/80

How well did you know this?
1
Not at all
2
3
4
5
Perfectly
146
Q

Cerebellar hematomas >_ cm in diameter will require surgical evacuation

A

3

Cerebellar hemorrhages should be urgently referred to neurosurgery service for evaluation

How well did you know this?
1
Not at all
2
3
4
5
Perfectly
147
Q

How is sacculary aneurysm managed?

A

> Lower the SBP to below 160 mmHg using nicardipine, labetalol, or esmolol. Cerebral perfusion pressure targeted to 60–70 mmHg.

> Treatment with the calcium channel antagonist nimodipine (60 mg PO every 4 h) improves outcome. prevents ischemia > vasospasm

> Euvolemia is targeted. Free-water restriction is contraindicated

> Pneumatic compression stockings applied to prevent PE.

> Unfractionated heparin administered subcutaneously for DVT prophylaxis can be initiated within 1–2 days following endovascular treatment or craniotomy

How well did you know this?
1
Not at all
2
3
4
5
Perfectly
148
Q

Headache symptoms that suggest a serious underlying condition

A
How well did you know this?
1
Not at all
2
3
4
5
Perfectly
149
Q

MOA of rimegepant

A

CGRP antagonists

How well did you know this?
1
Not at all
2
3
4
5
Perfectly
150
Q

When is triptan given in px with migraine

A

when aura is completed and headache has begun

How well did you know this?
1
Not at all
2
3
4
5
Perfectly
151
Q

What migraine prophylactic medication is contraindicated could aggravate parkinson’s disease

A

flunarizine

How well did you know this?
1
Not at all
2
3
4
5
Perfectly
152
Q

brain tumors has distinctive histopathologic feature of perinuclear clearing (giving rise to a “fried egg” appearance) and a reticular pattern of blood vessel growth?

A

Oligodendrogliomas

How well did you know this?
1
Not at all
2
3
4
5
Perfectly
153
Q

Mutation associated with oligodendroglioma

A

codeletion of 1p/19q

How well did you know this?
1
Not at all
2
3
4
5
Perfectly
154
Q

extrinsic brain tumors that arise from the dura mater and are completely composed of neoplastic meningothelial (arachnoid cap) cells

A

meningoma

How well did you know this?
1
Not at all
2
3
4
5
Perfectly
155
Q

Tumors with a strong propensity to metastasize
to axial skeleton

A

prostate CA, breast CA

also the 2 most common cause of epidural mets
Most common site: THORACIC

How well did you know this?
1
Not at all
2
3
4
5
Perfectly
156
Q

EEG finding in ATYPICAL absence seizure

A

EEG shows a generalized, slow spike- and-slow-wave pattern with a frequency of ≤2.5 per second

How well did you know this?
1
Not at all
2
3
4
5
Perfectly
157
Q

EEG finding in atonic seizure

A

Brief, generalized spike-and-wave discharges → immediately by diffuse slow waves

How well did you know this?
1
Not at all
2
3
4
5
Perfectly
158
Q

Which of the following is seen in patients with thalamic hemorrhage?

a. Pinpoint (1mm) pupils that are reactive to light.
b. Aphasia with preserved verbal repetition.
c. Heteronymous visual field defects due to interruption of the visual pathway.
d. Contralateral Horner’s syndrome and retraction nystagmus.

A

b. Aphasia with preserved verbal repetition.

a. Pinpoint (1mm) pupils that are reactive to light. –> unreactive to light

c. Heteronymous visual field defects due to interruption of the visual pathway.–> homonymous

d. Contralateral Horner’s syndrome and retraction nystagmus.–> ipsilateral

Thalamic hemorrhages also produce a contralateral hemiplegia or hemiparesis from pressure on, or dissection into, the adjacent internal capsule. A prominent sensory deficit involving all modalities is usually present.

Thalamic hemorrhages cause several typical ocular disturbances by extension inferiorly into the upper midbrain. These include deviation of the eyes downward and inward so that they appear to be looking at the nose, unequal pupils with absence of light reaction, skew deviation with the eye opposite the hemorrhage displaced downward and medially, ipsilateral Horner’s syndrome, absence of convergence, paralysis of vertical gaze, and retraction nystagmus.

Patients may later develop a chronic, contralateral pain syndrome (Déjérine-Roussy syndrome).

How well did you know this?
1
Not at all
2
3
4
5
Perfectly
159
Q

Which of the following drugs for migraine prophylaxis is correctly paired with its corresponding side effect?

a. Propranolol: hair loss
b. Flunarizine: drowsiness
c. Topiramate: weight gain
d. Valproic acid: weight loss

A

b. Flunarizine: drowsiness

a. Propranolol: hair loss–> not typical side effect
c. Topiramate: weight gain –> weight loss
d. Valproic acid: weight loss –> weight gain

How well did you know this?
1
Not at all
2
3
4
5
Perfectly
160
Q

Intracerebral hemorrhages at the subcortical regions are commonly seen in patients with

a. Metastatic brain tumors
b. Cerebral amyloid angiopathy
c. Uncontrolled hypertension
d. Capillary telangiectasia

A

c. Uncontrolled hypertension

ICH due to metastatic brain tumors, cerebral amyloid angiopathy and capillary telangiectasia are seen in the cortical areas of the brain.

How well did you know this?
1
Not at all
2
3
4
5
Perfectly
161
Q

What is the best strategy to reduce the burden of stroke?

a. Identification and control of modifiable risk factors, and especially hypertension and diabetes
b. Administration of anti-platelet therapy to patients with history of TIA or atherothrombotic stroke
c. Long-term vitamin K antagonists (VKAs) for preventing atherothrombotic stroke for either intracranial or extracranial cerebrovascular disease
d. Balloon angioplasty coupled with stenting to open stenotic carotid arteries and maintain their patency

A

a. Identification and control of modifiable risk factors, and especially hypertension and diabetes

How well did you know this?
1
Not at all
2
3
4
5
Perfectly
162
Q

Which of the following is incorrect regarding management of patients with coma?

a. An oropharyngeal airway is adequate to keep the pharynx open in a drowsy patient who is breathing normally.
b. Hypotension should be corrected slowly to avoid reperfusion injury.
c. Naloxone and dextrose are administered if narcotic overdose or hypoglycemia is a possibility.
d. Thiamine is given along with glucose to avoid provoking Wernicke’s encephalopathy in malnourished patients.

A

b. Hypotension should be corrected slowly to avoid reperfusion injury.

In the management of coma, hypotension is a medical emergency that must be corrected rapidly, as prolonged hypotension can lead to inadequate cerebral perfusion, worsening ischemic brain injury, and multi-organ failure.

How well did you know this?
1
Not at all
2
3
4
5
Perfectly
163
Q

Which is not a hallmark of acute Wernicke’s disease?
a. Confusion
b. Delirium
c. Impairment of eye movements
d. Gait ataxia

A

b. Delirium

How well did you know this?
1
Not at all
2
3
4
5
Perfectly
164
Q

What ethanol level in mmol/L in non-habituated patients generally causes impaired mental activity?

a. 43
b. 54
c. 65
d. 76

A

a. 43

a level of >65 mmol/L (0.3 g/dL) is associated with stupor.

The development of tolerance may allow some chronic alcoholics to remain awake at levels >87 mmol/L (0.4 g/dL).

How well did you know this?
1
Not at all
2
3
4
5
Perfectly
165
Q

A 45 year old male consulted due to recurrent headaches. On history, he had 2 previous hospital admissions due to intracerebral hemorrhages, as confirmed through plain Cranial CT scan. The last episode of intracerebral hemorrhage occurred 1 month prior to consult. What is the recommended cranial imaging technique for this patient?

a. Repeat Cranial CT Scan
b. Cranial MRI
c. 4 vessel angiogram
d. Transcranial doppler

A

b. Cranial MRI

In patients with subacute hemorrhage, MRI is the imaging of choice because the MR sequences may detect micro- hemorrhages, and determine the etiology of the bleed (eg. Cavernous malformations, amyloid, hemorrhagic metastaseds)

CT scan is recommended for ACUTE Intracerebral hemorrhage as it can be done faster than MRI, and is reliable in detecting acute blood.

4 vessel angiogram is indicated for patients suspected to have vascular malformations and aneurysms. Transcranial doppler is used to monitor vasospasm and cerebral bloodflow

How well did you know this?
1
Not at all
2
3
4
5
Perfectly
166
Q

A 62 year old male was brought to the ER due to sudden onset of occipital headache, vomiting, left sided weakness, followed by decrease sensorium. The patient was seen drowsy, with a BP of 210/100 and Cranial CT scan done showed a 35cc bleed in the basal ganglia with a 1 cm midline shift. Which of the following would be part of your management plan for this patient?
a. Maintain MAP 110 – 130
b. Keep head on low back rest
c. Start IV Hydralazine to reach target BP <160/90
d. Initiate osmotic agents as soon as possible

A

d. Initiate osmotic agents as soon as possible

This patient who has a 1cm midline shift on CT scan and is starting to have decreased sensorium should be started on osmotic agents such as Mannitol or hypertonic saline. Maintaining a MAP of 110-130 to facilitate permissive hypertension is done in the management of cerebral INFARCTION.

Recent trials in BP management for hypertensive ICH (INTERACT 2 AND ATTACH 2) showed that lowering the SBP to 140-180mmHg is likely safe and possibly beneficial.

Patients with ICH are placed in moderate to high back rest to improve venous return to the heart. In lowering BP, Nonvasodilating IV meds such as Nicardipine, Labetalol and Esmolol should be used instead of Hydralazine.

167
Q

A 35 year old male complained of sudden severe headache and vomiting. On PE, the patient is awake, oriented, with a right LR palsy, and no other sensorimotor deficits. Cranial CT Scan showed subarachnoid hemorrhage. What is this patient’s SAH Grade based on the Hunt-Hess Scale ?
a. 1
b. 2
c. 3
d. 4

A

b. 2

2= CN
3= motor
4= stu4
5= lima =coma

168
Q

After lifting a heavy object, a 40 year old male complained of sudden severe headache associated right sided eye pain. Upon shining a light on the right pupil, it was dilated, and non reactive to light. But upon shining a light on the left pupil, the contralateral pupillary reflex on the right was intact. Where is the most common likely site of the aneurysm?

a. Anterior Cerebral Artery
b. Inferior Cerebellar Artery
c. Middle Cerebral artery
d. Posterior Communicating Artery

A

d. Posterior Communicating Artery

An aneurysm of the posterior communicating artery will compress the third cranial nerve due to its proximity to the latter. This would result to an ipsilateral third cranial nerve is palsy, and present with pupillary dilation, and loss of ipsilateral (but retained contralateral) light reflex. Focal pain above or behind the eye, may occur with an expanding aneurysm at the junction of the posterior communicating artery and the internal carotid artery.

Aneurysms of the Anterior circulation (ACA or Acomm) would present with leg weakness. Aneurysms involving the posterior inferior cerebellar artery or anterior inferior cerebellar artery often present with occipital and posterior cervical pain. MCA aneurysms may present with pain in or behind the eye and in the low temple but this would not present with third nerve palsies.

169
Q

What is the ICH score of an 78 year old male with a 35cc bleed on the right capsuloganglionic area with intraventricular extension, and is GCS 12 on examination?
a. 2
b. 3
c. 4
d. 5

A

b. 3

thalamus is supratentorial

170
Q

A 19 year old male was brought to the ER due to a 2 week history of low grade fever, cough, night sweats and headache. On the day of admission, the patient had sudden left sided weakness and nape pain. What is the most likely etiology of the patient’s weakness?

a. Arteritis
b. Brain abscess
c. Empyema
d. Metastasis

A

a. Arteritis

This is a typical case of TB meningitis, with a subacute onset of symptoms and sudden focal deficit or stroke due to arteritis.

The focal deficits due to empyema, brain abscesses and metastasis usually present gradually.

171
Q

A 40 year old diabetic female was brought to the ER due to a 5 day history of fever and cough followed by a 3 day history of headache and increased sleeping in time. On PE, there was note of nuchal rigidity and positive Kernig’s sign. What is the most likely causative agent of this patient’s infection?
a. Neisseria meningitidis
b. Haemiphilus influenzae
c. Listeria monocytogenes
d. Streptococcus pnuemoniae

A

d. Streptococcus pnuemoniae

The patient initially presented with community acquired pneumonia which is the most important risk factor in developing pneumococcal meningitis. The organisms most often responsible for community-acquired bacterial meningitis are Streptococcus pneumoniae (~50%), Neisseria meningitidis (~25%), group B streptococci (~15%), and Listeria monocytogenes (~10%). Haemophilus influenzae type b accounts for <10% of cases of bacterial meningitis in most series.

There are a number of predisposing conditions that Additional risk factors include coexisting acute or chronic pneumococcal sinusitis or otitis media, alcoholism, diabetes, splenectomy, hypogammaglobulinemia, complement deficiency, and head trauma with basilar skull fracture and CSF rhinorrhea

172
Q

An 18 year old male was admitted and managed as a case of fungal meningitis. On the 7th day of treatment, the patient complained of bilateral lower extremity weakness, increased severity of headache and developed decrease in sensorium. What is the most likely cause of his symptoms?

a. Arteritis
b. Hydrocephalus
c. Status epilepticus
d. Spinal meningitis

A

b. Hydrocephalus

Lower extremity weakness with headache and decreased sensorium are signs most compatible with increased intracranial pressure.

In fungal meningitis, the most common complication is also hydrocephalus. Patients who develop hydrocephalus should receive a CSF diversion device. A ventriculostomy can be used until CSF fungal cultures are sterile, at which time the ventriculostomy is replaced by a ventriculoperitoneal shunt. Arteritis will present with focal neurologic deficits, while spinal meningitis will present with multiple radiculopathies. Status epilepticus would present with seizures

173
Q

A 23 year old female complained of tunneling of vision and gradually lost consciousness. She then had tonic movements of her extremities lasting 10 seconds, afterwhich, she was noted to be pale and disoriented for 3 minutes. What is the most likely diagnosis of this patient?
a. Pseudoseizure
b. Seizure
c. Syncope
d. Transient Ischemic Attack

A

c. Syncope

The patient’s premonitory symptom of tunneling of vision, gradual transition to unconsciousness and tonic movements lasting 10 seconds are features seen in syncope. Seizures usually present with immediate loss of consciousness lasting minutes, with tonic / clonic moevements lasting 30-60 seconds. Disorientation in seizures would last many minutes to hours while in syncope, it would last for less than 5 minutes. Patients with pseudoseizures present with side-to- side turning of the head, asymmetric and large-amplitude shaking movements of the limbs, twitching of all four extremities without loss of consciousness, and pelvic thrusting. In transient ischemic attack, there is usually a focal weakness that resolves in 1 hour with no evidence of infarction in imaging.

174
Q

Which statement is true regarding intracerebral hemorrhage

a. Hypertensive hemorrhages usually result from spontaneous rupture of superficial arteries in the brain
b. In elderly nonhypertensive patients, cerebral amyloid angiopathy should be considered
c. Medium sized arteries in the basal ganglia are most prone to hypertension induced vascular injury
d. Most intracerebral hemorrhages would have significant hematoma expansion after 48 hours

A

b. In elderly nonhypertensive patients, cerebral amyloid angiopathy should be considered –> most common cause of lobar hemorrhage in elderly

Why other choices are wrong
a. Hypertensive hemorrhages usually result from spontaneous rupture of superficial arteries in the brain –> deep seated arteries

c. Medium sized arteries in the basal ganglia are most prone to hypertension induced vascular injury –> small penetrating arteries

d. Most intracerebral hemorrhages would have significant hematoma expansion after 48 hours –> 24h

175
Q

Patients with primary white matter disorders often present with which of the following?
a. Seizures
b. Early cognitive decline
c. Movement disorders
d. Long tract signs

A

d. Long tract signs

The gray matter is composed predominantly of neuronal cell bodies. Hence disorders in the grey matter would often present with seizures, early cognitive decline and movement disorders.

On the other hand, the white matter is composed of axons, their myelin and glial cells which transmit signals to the other cells in the central nervous system. In white matter disorders, these signals are interrupted hence long track signs involving motor, sensory and cerebellar pathways are seen in these patients.

176
Q

Which is true in the management of acute subarachnoid hemorrhage?

a. Immediate referral to a neurosurgeon or endovascular surgeon is recommended for all patients
b. Treatment with Nimodipine 30mg tab q4 is given to reduce vasospasm and prevent ischemic injury
c. Glucocorticoids are routinely recommended to reduce cerebral edema
d. Free water restriction is useful in patients at risk for delayed cerebral ischemia

A

a. Immediate referral to a neurosurgeon or endovascular surgeon is recommended for all patients

The definitive management to prevent rerupture of aneurysm is through “clipping” by a neurosurgeon or “coiled” by an endovascular surgeon. These patients should be referred immediately since early aneurysm repair prevents rerupture and allows the safe application of techniques to improve blood flow in the brain.

Treatment with the calcium channel antagonist nimodipine (60 mg or 2 tabs of 30mg q4) improves outcome, perhaps by preventing ischemic injury rather than reducing the risk of vasospasm.

Glucocorticoids may help reduce the head and neck ache caused by the irritative effect of the subarachnoid blood. However, there is no good evidence that they reduce cerebral edema, are neuroprotective, or reduce vascular injury, and their routine use therefore is not recommended.

Free-water restriction is contraindicated in patients with SAH at risk for DCI because hypovolemia and hypotension may occur and precipitate cerebral ischemia.

177
Q

Which of the following is true among patients with intracerebral hemorrhage and coagulopathies?

a. All patients on antiplatelet drugs should be transfused with platelet concentrate
b. Administration of 1 dose of Idarizucimab in patients on Dabigatran quickly reverses its anticoagulation effect
c. Recombinant factor VIIa is a recommended treatment to improve clinical outcomes in these patients
d. Thrombocytopenia should be corrected with platelet transfusion

A

d. Thrombocytopenia should be corrected with platelet transfusion

Platelet transfusions should only be given in patients on antiplatelets if these patients present with thrombytopenia. In a recent trial, for ICH patients on antiplatelets but without thrombocytopenia, platelet transfusion suggested no benefit and was noted to have possible harm

Administration of two doses of Idarizucimab reverses the anticoagulation effect of dabigatran quickly.

A phase 3 trial of treatment with recombinant factor VIIa reduced hematoma expansion in ICH patients. However, clinical outcomes were not improved, so use of this drug is not recommended.

When ICH is associated with thrombocytopenia (platelet count <50,000/μL), transfusion of fresh platelets is indicated

178
Q

Which of the following is true in the management of patients with HSV encephalitis?
a. Intravenous Acyclovir at 20mg/kg q8 for 21 days in adults
b. Oral Valacyclovir is given as a supplemental therapy after IV acyclovir is given
c. IV acyclovir should be diluted at a concentration 7mg/mL and infused over 30 minutes
d. Seizure prophylaxis should be considered in severe cases of encephalitis

A

d. Seizure prophylaxis should be considered in severe cases of encephalitis

In the treatment of HSV encephalitis, IV Acyclovir is given at 10mg/kg q8 for 21 days in adults. Neonates are given 20mg/kg IV Acvclovir q8.

Oral Valacyclovir has not been evaluated as a supplemental therapy after IV acyclovir is given in patients with HSV encephalitis.

IV acyclovir should be diluted at a concentration <7mg/mL prior to infusion and slowly infused over 1 hour to prevent renal dysfunction.

Because of the high frequency of seizures in severe cases of encephalitis, seizure prophylaxis should be considered in severe cases of encephalitis

179
Q

Difference between fatigability vs fatigue

A

Fatigability of muscle power is a cardinal manifestation of some neuromuscular disorders such as myasthenia gravis and is distinguished from fatigue by finding clinically evident diminution of the amount of force that a muscle generates upon repeated contraction

180
Q

What is breakaway weakness

A

On confrontational testing, full power may be generated for only a brief period before the patient suddenly gives way to the examiner. This type of weakness is often referred to as breakaway weakness and may or may not be associated with pain.

This is contrasted with weakness due to lesions in the motor tracts or lower motor unit, in which the patient’s resistance can be overcome in a smooth and steady fashion and full power can never be generated.

181
Q

Differentiate LMN vs UMN weakness

A

Fasciculations –> LMN

182
Q

Pure motor hemiparesis is seen in lesions of what structures?

A

A “pure motor” hemiparesis of the face, arm, and leg often is due to a small, discrete lesion in the posterior limb of the internal capsule, cerebral peduncle in the midbrain, or upper pons.

183
Q

In obtunded patients, evaluation begins with a CT or MRI scan of the brain. If upper motor neuron signs are present but the patient is alert, the initial test is usually an MRI of the ________

A

cervical cord.

184
Q

How do you differentiate proximal weakness from myopathy vs NMJ vs anterior horn

A

Myopathy often produces symmetric weakness of the pelvic or shoulder girdle muscles.

Diseases of the neuromuscular junction, such as myasthenia gravis, may present with symmetric proximal weakness often associated with ptosis, diplopia, or bulbar weakness and fluctuate in severity during the day.

In anterior horn cell disease, proximal weakness is usually asymmetric, but it may be symmetric especially in genetic forms. Numbness does not occur with any of these diseases.

The evaluation usually begins with determination of the serum creatine kinase level and electrophysiologic studies.

185
Q

most useful bedside test of peripheral vestibular function

A

The most useful bedside test of peripheral vestibular function is the head impulse test, in which the vestibulo-ocular reflex (VOR) is assessed with small-amplitude (~20 degrees) rapid head rotations.

While the patient fixates on a target, the head is rotated quickly to the left or right. If the VOR is deficient, the rotation is followed by a catch-up saccade in the opposite direction after a rightward rotation).

The head impulse test can identify both unilateral (catch-up saccades after rotations toward the weak side) and bilateral (catch-up saccades after rotations in both directions) vestibular hypofunction

186
Q

Nystagmus seen in posterior canal BPPV

A

Posterior canal BPPV can be diagnosed confidently if transient upbeating-torsional nystagmus is seen.

For posterior canal BPPV, the Epley maneuver is the most commonly used procedure

186
Q

Two forms of nystagmus that are characteristic of lesions of the cerebellar pathways

A

Two forms of nystagmus that are characteristic of lesions of the cerebellar pathways are vertical nystagmus with downward fast phases (downbeat nystagmus) and horizontal nystagmus that changes direction with gaze (gazeevoked nystagmus)

187
Q

Treatment for Menieres dse

A

Ménière’s disease is associated with excess endolymph fluid in the inner ear; hence the term endolymphatic hydrops.

Patients suspected of having Ménière’s disease should be referred to an otolaryngologist for further evaluation.

Diuretics and sodium restriction are typically the initial treatments. If attacks persist, injections of glucocorticoids or gentamicin into the middle ear may be considered.

Nonablative surgical options include decompression and shunting of the endolymphatic sac. Full ablative procedures (vestibular nerve section, labyrinthectomy) are seldom required

188
Q

Most common cause of syncope

A

In population-based studies, neurally mediated syncope is the most common cause of syncope.

2nd most common: cardiac

189
Q

High-Risk Features Indicating Hospitalization or Intensive Evaluation of Syncope

A
190
Q

Cerebral blood flow in ml/min

A

Typically, cerebral blood flow ranges from 50–60 mL/min per 100 g brain tissue and remains relatively constant over perfusion pressures ranging from 50–150 mmHg

191
Q

2 types of syncope based on efferent pathway

A

Vasodepressor syncope describes syncope predominantly due to efferent, sympathetic, vasoconstrictor failure; cardioinhibitory syncope describes syncope predominantly associated with bradycardia or asystole due to increased vagal outflow;a

192
Q

cornerstones of the management of neurally mediated syncope

A

Reassurance, education, avoidance of provocative stimuli, and plasma volume expansion with fluid and salt are the cornerstones of the management of neurally mediated syncope. Isometric counterpressure maneuvers of the limbs (tensing of the abdominal and leg muscles, handgrip and arm tensing, and leg crossing) may raise blood pressure by increasing central blood volume and cardiac output. Of these, abdominal muscle tensing is the most effective

193
Q

T/F ALL patients with tonic clonic movements have generalized seizures

A

. Whereas tonic-clonic movements are the hallmark of a generalized seizure, myoclonic and other movements also may occur in up to 90% of syncopal episodes.

194
Q

How do you differentiate LOC from seizure vs syncope?

A

Loss of consciousness associated with a seizure usually lasts >5 min and is associated with prolonged postictal drowsiness and disorientation, whereas reorientation occurs almost immediately after a syncopal event.

195
Q

What is cataplexy?

A

Patients with cataplexy experience an abrupt partial or complete loss of muscular tone triggered by strong emotions, typically anger or laughter. Unlike syncope, consciousness is maintained throughout the attacks, which typically last between 30 s and 2 min. There are no premonitory symptoms. Cataplexy occurs in 60%–75% of patients with narcolepsy

196
Q

Diagnostic step that should be performed in patients who have experienced syncope during or shortly after exercise

A

Treadmill exercise testing with ECG and blood pressure monitoring should be performed in patients who have experienced syncope during or shortly after exercise. T

197
Q

How is vibratory sense tested?

A

The sense of vibration is tested with an oscillating tuning fork that vibrates at 128 Hz. Vibration is tested over bony points, beginning distally; in the feet, it is tested over the dorsal surface of the distal phalanx of the big toes and at the malleoli of the ankles, and in the hands, it is tested dorsally at the distal phalanx of the fingers. If abnormalities are found, more proximal sites should be examined. Vibratory thresholds at the same site in the patient and the examiner may be compared for control purposes.

198
Q

Inability to recognize numbers or letters is termed ______

A

Inability to recognize numbers or letters is termed agraphesthesia

Stereognosis refers to the ability to identify common objects by palpation, recognizing their shape, texture, and size.

199
Q

PE findings in Brown-Séquard syndrome

A

Lateral hemisection of the spinal cord produces the Brown-Séquard syndrome, with absent pain and temperature sensation contralaterally and loss of proprioceptive sensation and power ipsilaterally below the lesion

200
Q

Treatment for dyesthesias

A

Dysesthesias, when severe and persistent, may respond to anticonvulsants (carbamazepine, 100–1000 mg/d; gabapentin, 300–3600 mg/d; or pregabalin, 50–300 mg/d), antidepressants (amitriptyline, 25–150 mg/d; nortriptyline, 25–150 mg/d; desipramine, 100–300 mg/d; or venlafaxine, 75–225 mg/d).

201
Q

The most common cause of frontal gait disorder is

A

vascular disease, particularly subcortical small-vessel disease in the deep frontal white matter and centrum ovale

202
Q

Differentiate cerebellar vs sensory ataxia vs frontal gait

A
203
Q

Steppage gait is seen in what condition/s

A

With distal weakness (peripheral neuropathy), the step height is increased to compensate for foot drop, and the sole of the foot may slap on the floor during weight acceptance, termed the steppage gait.

204
Q

Duration of TUG test result to be classified as high risk for fall

A

The Timed Up and Go (“TUG”) test involves timing a patient as they stand up from a chair, walk 10 feet, turn, and then sit down. Patients with a history of falls or those requiring >12 s to complete the TUG test are at high risk for falls and should undergo further assessment

205
Q

What martial art program as been demonstrated to reduce the risk of falls and injury in patients with Parkinson’s disease?

A

A Tai Chi exercise program has been demonstrated to reduce the risk of falls and injury in patients with Parkinson’s disease

206
Q

Hallmark of delirium

A

The hallmark of delirium is a deficit of attention, although all cognitive domains—including memory, executive function, visuospatial tasks, and language—are variably involved.

The attentional deficit that serves as the neuropsychological hallmark of delirium has a diffuse localization within the brainstem, thalamus, prefrontal cortex, and parietal lobes

206
Q

The two most consistently identified risk factors for delirium are

A

The two most consistently identified risk factors are older age and baseline cognitive dysfunction.

207
Q

Basic tests that must be requested for ALL px with delirium

A

Basic screening labs, including a complete blood count, electrolyte panel, and tests of liver and renal function, should be obtained in all patients with delirium.

In elderly patients, screening for systemic infection, including chest radiography, urinalysis and culture, and possibly blood cultures, is important.

In younger individuals, serum and urine drug and toxicology screening may be appropriate earlier in the workup

Multiple studies have demonstrated that brain imaging in patients with delirium is often unhelpful.

207
Q

Cerebrovascular etiologies of delirium are usually due to _______

A

global hypoperfusion in the setting of systemic hypotension from heart failure, septic shock, dehydration, or anemia

Focal strokes in the right parietal lobe and medial dorsal thalamus rarely can lead to a delirious state.

207
Q

Personality change, disinhibition, and weight gain or compulsive eating suggest what type of dementia

A

Frontotemporal dementia

208
Q

Routine tests for px with dementia

A

Brain imaging routine for dementia but not delirium

209
Q

Visual hallucinations, REM sleep behavior disorder, delirium, Capgras syndrome, parkinsonism are seen in what dementia

A

Dementia with Lewy bodies

Visual –> lewy bodies

210
Q

Screening tool/s that is/are sensitive for early stage dementia

A

None

Brief screening tools such as the Mini-Mental State Examination (MMSE), the Montreal Cognitive Assessment (MOCA), the Tablet Based Cognitive Assessment Tool, and Cognistat can be used to capture dementia and follow progression. None of these tests is highly sensitive to early-stage dementia or reliably discriminates between dementia syndromes.

211
Q

Earliest deficits on cognitive testing in px with Alzheimers and frontotemporal dementia

A

In AD, the early deficits involve episodic memory, category generation (“name as many animals as you can in 1 minute”), and visuoconstructive ability. Usually deficits in verbal or visual episodic memory are the first neuropsychological abnormalities detected, and tasks that require the patient to recall a long list of words or a series of pictures after a predetermined delay will demonstrate deficits in most patients.

In FTD, the earliest deficits on cognitive testing involve executive control or language (speech or naming) functions, but some patients lack either finding despite profound social-emotional deficits.

212
Q

Akinetic mutism is seen in what lesions?

A

Akinetic mutism refers to a partially or fully awake state in which the patient remains virtually immobile and mute but can form impressions and think, as demonstrated by later recounting of events. This condition results from damage in the regions of the medial thalamic nuclei or the frontal lobes (particularly lesions situated deeply or on the orbitofrontal surfaces) or from extreme hydrocephalus.

213
Q

Hypothermia itself causes coma when the temperature is <____ C regardless of the underlying etiology

A

31°C (87.8°F)

213
Q

1st sign of uncal herniation

A

The uncus can compress the third nerve as the nerve traverses the subarachnoid space, causing enlargement of the ipsilateral pupil as the first sign

214
Q

In a drowsy and confused patient, bilateral asterixis is a sign of

A

metabolic encephalopathy or drug intoxication.

215
Q

Bilaterally dilated and non reactive pupils is seen in lesions of?

A

The most extreme pupillary sign, bilaterally dilated and unreactive pupils, indicates severe midbrain damage, usually from compression by a supratentorial mass.

216
Q

Reactive and bilaterally small (1–2.5 mm) but not pinpoint pupils are seen in _______

A

Reactive and bilaterally small (1–2.5 mm) but not pinpoint pupils are seen in metabolic encephalopathies or in deep bilateral hemispheral lesions such as hydrocephalus or thalamic hemorrhage.

Even smaller reactive pupils (<1 mm) characterize opioid overdoses but also occur with extensive pontine hemorrhage.

217
Q

Ocular bobbing is seen in ________

A

Ocular bobbing” describes brisk downward and slow upward movements of the eyes associated with loss of horizontal eye movements and is diagnostic of bilateral pontine damage, usually from thrombosis of the basilar artery.

“Ocular dipping” is a slower, arrhythmic downward movement followed by a faster upward movement in patients with normal reflex horizontal gaze; it usually indicates diffuse cortical anoxic damage.

218
Q

Alpha coma is seen in _____

A

Alpha coma results from pontine or diffuse cortical damage and is associated with a poor prognosis.

219
Q

extreme delta brush on EEG is characteristic of ______

A

A unique EEG pattern in adults of “extreme delta brush” is characteristic of a specific (anti–N-methyl-d-aspartate [NMDA] receptor) form of autoimmune encephalitis.

220
Q

Characteristics of brain dead px

A

Established criteria contain two essential elements, after assuring that no confounding factors (e.g., hypothermia, drug intoxication) are present: (1) widespread cortical destruction that is reflected by deep coma and unresponsiveness to all forms of stimulation; and (2) global brainstem damage as demonstrated by absent pupillary light reaction, absent corneal reflexes, loss of oculovestibular reflexes, and destruction of the medulla, manifested by complete and irreversible apnea.

Demonstration that apnea is due to medullary damage requires that the Pco2 be high enough to stimulate respiration during a test of spontaneous breathing. Apnea testing can be done by the use of preoxygenation with 100% oxygen prior to and following removal of the ventilator. CO2 tension increases ~0.3–0.4 kPa/min (2–3 mmHg/min) during apnea. Apnea is confirmed if no respiratory effort has been observed in the presence of a sufficiently elevated Pco2 . The apnea test is usually stopped if there is cardiovascular instability and alternative means of testing can be employed

221
Q

T/F Metabolic comas have a far better prognosis than traumatic ones.

A

True

221
Q

For anoxic coma, clinical signs such as the pupillary and motor responses after ___day, ____ days, and ___ week/s have predictive value

A

For anoxic coma, clinical signs such as the pupillary and motor responses after 1 day, 3 days, and 1 week have predictive value

1,3,7

222
Q

Specific loss of the orexin neurons produces the sleep disorder _____

A

narcolepsy

223
Q

Subjects awakened from NREM/REM sleep recall vivid dream imagery >80% of the time?

A

Subjects awakened from REM sleep recall vivid dream imagery >80% of the time, especially later in the night. Less vivid imagery may also be reported after NREM sleep interruptions

REM- REMember dreams

224
Q

BP and HR decreases during what phase of sleep

A

Blood pressure and heart rate decrease during NREM sleep, particularly during N3 sleep.

225
Q

HLA that is found in >90% of people with type 1 narcolepsy

A

Certain human leukocyte antigens (HLAs) can increase the risk of autoimmune disorders, and narcolepsy has the strongest known HLA association.

HLA DQB1*06:02 is found in >90% of people with type 1 narcolepsy, whereas it occurs in only 12–25% of the general population.

Researchers now hypothesize that in people with DQB1*06:02, an immune response against influenza, Streptococcus, or other infections may also damage the orexin-producing neurons through a process of molecular mimicry.

226
Q

Treatment for cataplexy

A

Cataplexy is usually much improved with antidepressants that increase noradrenergic or serotonergic tone because these neurotransmitters strongly suppress REM sleep and cataplexy.

Venlafaxine (37.5–150 mg each morning) and fluoxetine (10–40 mg each morning) are often quite effective. The tricyclic antidepressants, such as protriptyline (10–40 mg/d) or clomipramine (25–50 mg/d) are potent suppressors of cataplexy, but their anticholinergic effects, including sedation and dry mouth, make them less attractive

Sodium oxybate, twice each night, is also very helpful in reducing cataplexy.

227
Q

Treatment for narcolepsy

A

The treatment of narcolepsy is symptomatic. Most patients with narcolepsy feel more alert after sleep, and they should be encouraged to get adequate sleep each night and to take a 15- to 20-min nap in the afternoon. This nap may be sufficient for some patients with mild narcolepsy, but most also require treatment with wakepromoting medications.

Modafinil is often used because it has fewer side effects than amphetamines and a relatively long halflife; for most patients, 200–400 mg each morning is very effective.

Methylphenidate (10–20 mg bid) or dextroamphetamine (10 mg bid) are also effective, but sympathomimetic side effects, anxiety, and the potential for abuse can be concerns.

Solriamfetol, a norepinephrine– dopamine reuptake inhibitor (75–150 mg daily), and pitolisant, a selective histamine 3 (H3 ) receptor antagonist (8.9–35.6 mg daily), also improve sleepiness and have relatively few side effects.

Sodium oxybate (gamma hydroxybutyrate), given at bedtime and 3–4 h later, is often very valuable in improving alertness, but it can produce excessive sedation, nausea, and confusion

228
Q

Most common treateble cause of restless leg syndrome

A

IDA

iron replacement should be considered if the ferritin level is <75 ng/mL.

RLS is treated by addressing the underlying cause such as iron deficiency if present. Otherwise, treatment is symptomatic, and dopamine agonists or alpha-2-delta calcium channel ligands are used most frequently. Agonists of dopamine D2/3 receptors such as pramipexole (0.25–0.5 mg q7PM) or ropinirole (0.5–4 mg q7PM) are usually quite effective, but about 25% of patients taking dopamine agonists develop augmentation, a worsening of RLS such that symptoms begin earlier in the day and can spread to other body regions

229
Q

single most common finding in aphasic patients

A

A deficit of naming (anomia) is the single most common finding in aphasic patients.

Anomic aphasia is the single most common language disturbance seen in head trauma, metabolic encephalopathy, and Alzheimer’s disease.

230
Q

lesion site most commonly associated with Wernicke’s aphasia caused by CVAs

A

The lesion site most commonly associated with Wernicke’s aphasia caused by CVAs is the posterior portion of the language network. An embolus to the inferior division of the middle cerebral artery (MCA), to the posterior temporal or angular branches in particular, is the most common etiology

231
Q

The most common etiology of pure alexia

A

The most common etiology of pure alexia is a vascular lesion in the territory of the posterior cerebral artery or an infiltrating neoplasm in the left occipital cortex that involves the optic radiations as well as the crossing fibers of the splenium. Because the posterior cerebral artery also supplies medial temporal components of the limbic system, a patient with pure alexia also may experience an amnesia, but this is usually transient because the limbic lesion is unilatera

232
Q

What constitutes Gertsmann syndrome?

A

The combination of acalculia (impairment of simple arithmetic), dysgraphia (impaired writing), finger anomia (an inability to name individual fingers such as the index and thumb), and right-left confusion (an inability to tell whether a hand, foot, or arm of the patient or examiner is on the right or left side of the body) is known as Gerstmann’s syndrome.

MCA involvement

233
Q

The individual fragments of information remain preserved despite the limbic lesions and can sustain what is known as _____ memory

A

The individual fragments of information remain preserved despite the limbic lesions and can sustain what is known as implicit memory. For example, patients with amnestic states can acquire new motor or perceptual skills even though they may have no conscious knowledge of the experiences that led to the acquisition of these skills

234
Q

Bare minimum sites to be tested for reflexes

A

Biceps, achilles, patellar

235
Q

Reflexes to test for suspected injury to spinal cord or lumbosacral roots

A

Cremasteric (L1-L2)
Anal (S2-S4)

236
Q

Most common complication of myelography

A

Postural headache due to leakage of CSF

If persists >48h, place epidural blood patch

237
Q

Most feared complication of catheter angiography

A

Stroke (embolism/thrombosis)

238
Q

Most common seizure type from metabolic derangement

A

GTC

239
Q

Which of the following statements about intracranial meningitis is true?
a. CSF is produced by the arachnoid villi and is absorbed by the choroid plexus.
b. In basal meningitis, inflammatory deposits in the brainstem result to hydrocephalus
c. Stimulation of the nociceptive fibers in the brain parenchyma result to headache.
d. Raised intracranial pressure may present with palsy of the sixth cranial nerve.

A

d. Raised intracranial pressure may present with palsy of the sixth cranial nerve.

CSF is produced by the choroid plexus and absorbed by the arachnoid villi.

In basal meningitis, inflammatory deposits in the brainstem result to multiple cranial nerve palsies, not hydrocephalus. Stimulation of the nociceptive fibers in the meninges (not in the brain parenchyma) result to headache.

Raised intracranial pressure may present with palsy of the sixth cranial nerve. Obstruction of CSF pathways at the foramina or arachnoid villi may produce hydrocephalus and signs and symptoms of raised intracranial pressure (ICP), including headache, vomiting, apathy or drowsiness, gait instability, papilledema, visual loss, impaired upgaze, or palsy of the sixth cranial nerve.

240
Q

Which of the following increases the mortality risk in patients with bacterial meningitis?

a. Increased CSF glucose
b. Seizures after 24 hours of admission
c. Age >50
d. Decreased CSF protein

A

c. Age >50

In general, the risk of death from bacterial meningitis increases with
(1) decreased level of consciousness on admission,
(2) onset of seizures within 24 h of admission,
(3) signs of increased ICP,
(4) young age (infancy) and age >50,
(5) the presence of comorbid conditions including shock and/or the need for mechanical ventilation, and
(6) delay in the initiation of treatment.

Decreased CSF glucose concentration (<2.2 mmol/L [<40 mg/dL]) and markedly increased CSF protein concentration (>3 g/L [> 300 mg/dL]) have been predictive of increased mortality.

241
Q

Which of the following is typical of the CSF findings in most cases of viral encephalitis?

a. Increased glucose
b. Normal protein
c. CSF pleocytosis
d. RBC > 500 u/L

A

c. CSF pleocytosis

The characteristic CSF findings in viral encephalitis are: normal glucose and mildly elevated protein concentration. A CSF pleocytosis (>5 cells/μL) occurs in >95% of immunocompetent patients with documented viral encephalitis.

Only about 20% of patients with encephalitis will have a significant number of red blood cells (>500/μL) in the CSF in a nontraumatic tap. The pathologic correlate of this finding may be a hemorrhagic encephalitis of the type seen with HSV.

242
Q

What comprises the Cushing reflex of increased intracranial pressure?
a. Tachycardia, Hypertension and irregular respirations
b. Tachycardia, Hypotension and irregular respirations
c. Bradycardia, Hypertension and irregular respirations
d. Bradycardia, Hypotension and irregular respirations

A

c. Bradycardia, Hypertension and irregular respirations

243
Q

Which of the following CSF findings is seen in patients with bacterial meningitis?
a. Protein <45mg/dL
b. CSF/serum glucose <0.4
c. Glucose >2.2 mmol/L
d. Opening pressure <180 mmH2Oa

A

b. CSF/serum glucose <0.4

244
Q

Which of the following is the most common symptom accompanying severe migraine attacks?

a. Lightheadedness
b. Nausea
c. Photophopia
d. Vomiting

A

b. Nausea

245
Q

In patients with Bell’s palsy, what treatment strategy has been shown to modestly shorten recovery period and improve clinical outcome?

a. Acyclovir 800mg 5 times daily for 10 days
b. Artificial tears and massage of weakened muscle
c. Prednisone 60 – 80 mg daily for the first 5 days
d. Valacyclovir 100mg daily for 5-7 days

A

c. Prednisone 60 – 80 mg daily for the first 5 days

A course of glucocorticoids, given as prednisone 60–80 mg daily during the first 5 days and then tapered over the next 5 days, modestly shortens the recovery period and improves the functional outcome in Bell’s palsy. Although large and well-controlled randomized trials found no added benefit of the antiviral agents valacyclovir (1000 mg daily for 5–7 days) or acyclovir (400 mg five times daily for 10 days) compared to glucocorticoids alone, some earlier data suggested that combination therapy with prednisone plus valacyclovir might be marginally better than prednisone alone, especially in patients with severe clinical presentations

246
Q

A 64 year-old male was brought to the clinic because his granddaughter noticed the patient does not appear to notice the left side of his body. He would repeatedly bump that side, and when he attempted to shave, he only did so on his right side. What is the most likely artery blocked?
a. Pre-communal (A1) anterior cerebral artery
b. Post-communal (A1) anterior cerebral artery
c. Inferior division of the middle cerebral artery
d. Superior division of the middle cerebral artery

A

c. Inferior division of the middle cerebral artery

Proximal superior division – supplies large portions of the frontal and parietal cortices –> combination of sensory disturbance, motor weakness, and nonfluent aphasia

Inferior division of the MCA - supplies posterior part (temporal cortex) of the dominant hemisphere –> fluent (Wernicke’s) aphasia without weakness Jargon speech and an inability to comprehend written and spoken language are prominent features, often accompanied by a contralateral, homonymous superior quadrantanopia.

Inferior division of the MCA in the nondominant hemisphere –> hemineglect or spatial agnosia without weakness

247
Q

Triad of Lennox Gestaut syndrome

A

Multiple seizure types
EEG slowing <3 Hz
Impaired cognitive function

248
Q

Which artery is blocked if a patient manifests with contralateral hemiplegia, hemianesthesia (hypesthesia), and homonymous hemianopia?
a. Anterior choroidal artery
b. Basilar artery
c. Lenticulostriate vessels
d. Posterior cerebral artery

A

a. Anterior choroidal artery

Complete syndrome of anterior choroidal artery occlusion consists of:
- contralateral hemiplegia
- hemianesthesia (hypesthesia)
- homonymous hemianopia.

** similar to complete MCA occlusion except for lack of dysarthria/aphasia

However, because this territory is also supplied by penetrating vessels of the proximal MCA and the posterior communicating and posterior choroidal arteries, minimal deficits may occur, and patients frequently recover substantially.

249
Q

What pathophysiologic mechanism underlying brain ischemia is NOT accurately stated?
a. Small vessel stroke is due to occlusion either by atherothrombotic disease at its origin or by the development of lipohyalinotic thickening of the small branches of MCA, basilar, and vertebral arteries.
b. If low cerebral blood flow persists for a longer duration, infarction in the border zones between the major cerebral artery distributions may develop.
c. Cerebral ischemia is caused by a reduction in blood flow that lasts longer than several seconds. Neurologic symptoms are manifest within seconds because neurons lack glycogen, so energy failure is rapid.
d. Local thrombus formation rather than artery-to-artery embolism appears to be the dominant vascular mechanism causing large-vessel brain ischemia.

A

d. Local thrombus formation rather than artery-to-artery embolism appears to be the dominant vascular mechanism causing large-vessel brain ischemia.

Unlike the myocardial vessels, artery-to-artery embolism, rather than local thrombosis, appears to be the dominant vascular mechanism causing large-vessel brain ischemia.

250
Q

While there are no reliable clinical findings that conclusively separate cerebral ischemia from hemorrhage, which sign suggests ischemia as the more likely etiology?
a. Deficit that is maximal at onset, or remits
b. Higher initial blood pressure
c. More depressed level of consciousness
d. Worsening of symptoms after onset

A

a. Deficit that is maximal at onset, or remits

Perform an emergency non-contrast head CT scan to differentiate between ischemic stroke and hemorrhagic stroke

there are no reliable clinical findings that conclusively separate ischemia from hemorrhage, although a more depressed level of consciousness, higher initial blood pressure, or worsening of symptoms after onset favor hemorrhage, and a deficit that is maximal at onset, or remits, suggests ischemia.

251
Q

What should not be done if a patient is to be given intravenous recombinant tissue plasminogen activator (rtPA) for acute ischemic stroke?

a. Infuse through central line
b. Avoid urethral catheterization for at least 2 hours.
c. Give cryoprecipitate if there is decline in neurologic status during or after infusion.
d. Avoid giving other antithrombotic treatment for 24 hours

A

a. Infuse through central line

Peripheral line is sufficient

252
Q

What is the clinical outcome in bilateral infarction in the distal posterior cerebral arteries?
a. Cortical blindness but patient may deny it (Anton’s syndrome)
b. Third nerve palsy with contralateral ataxia (Claude’s syndrome)
c. Palinopsia (Balint’s syndrome)
d. Pupil is mydriatic with contralateral hemiplegia (Weber’s syndrome)

A

a. Cortical blindness but patient may deny it (Anton’s syndrome)

C also correct??

253
Q

In a patient with impending and early status epilepticus, which medication is NOT part of the initial therapeutics?
a. Levetiracetam
b. Midazolam
c. Pentobarbital
d. Phenytoin

A

c. Pentobarbital (PTB in picture)

Given in late refractory SE

254
Q

A woman who has had epilepsy for 5 years found out she’s 8 weeks pregnant. She had an episode of status epilepticus 4 months ago and is on valproic acid and phenytoin. Which is NOT a consideration in her management?
a. She will most likely have a complicated gestation and deliver a baby with at least mild physical or developmental defects.
b. She should be maintained on these effective drug therapy despite her being in the first trimester.
c. She should start taking folate (4 mg/d).
d. She will benefit from being treated with oral vitamin K (20 mg/d) in the last 2 weeks of pregnancy

A

a. She will most likely have a complicated gestation and deliver a baby with at least mild physical or developmental defects.

Most women with epilepsy who become pregnant will have an UNcomplicated gestation and deliver a normal baby

Seizure frequency during pregnancy: unchanged in ~50% of women, increase in ~30%, and decrease in ~20%

Enzyme-inducing drugs such as phenytoin cause a transient and reversible deficiency of vitamin K–dependent clotting factors in ~50% of newborn infants.

Although neonatal hemorrhage is uncommon, the mother should be treated with oral vitamin K (20 mg/d, phylloquinone) in the last 2 weeks of pregnancy, and the infant should receive intramuscular vitamin K (1 mg) at birth.

255
Q

A veteran told a story about how his comrade was tortured by having him drink gallons of water until he started convulsing. What was his comrade’s most probable serum sodium then?
a. 130-135 mmol/L
b. 126-130 mmol/L
c. 120-125 mmol/L
d. 115-119 mmol/L

A

d. 115-119 mmol/L

Coma and seizures are common accompaniments of large shifts in sodium and water balance in the brain. Sodium levels <125 mmol/L induce confusion, and levels <119 mmol/L are typically associated with coma and convulsions, especially when these levels are achieved quickly.

256
Q

On neurologic examination of a 52-year old male referred to for evaluation, the following were noted: Patient’s eyes were closed but woke up to name-calling. He was asked to extend his arms and dorsiflex the wrists and bilateral asterixis (flapping of the wrists) was seen. An accurate conclusion is there is a sign of _____?
a. Decerebrate rigidity
b. Metabolic encephalopathy
c. Middle cerebral artery infarction
d. Seizure

A

b. Metabolic encephalopathy

In a drowsy and confused patient, bilateral asterixis is a sign of metabolic encephalopathy or drug intoxication.

257
Q

What is NOT a common cause of generalized convulsive status epilepticus ?
a. Lack of sleep
b. Metabolic disturbances
c. Trauma to the head
d. Withdrawal of anticonvulsant

A

a. Lack of sleep

Precipitant only not a cause

The most common causes of GCSE are anticonvulsant withdrawal or noncompliance, metabolic disturbances, drug toxicity, CNS infection, CNS tumors, refractory epilepsy, and head trauma.

Sleep deprivation can be a precipitant though, including psychological or physical stress and hormonal changes as well.

258
Q

Which statement is correct regarding different states of reduced alertness?
a. Patients in the vegetative state especially from posttraumatic cases have no meaningful cerebral activation in response to verbal and other stimuli.
b. Stuporous patients can be transiently awakened by vigorous stimuli and avoid or withdraw from uncomfortable or aggravating stimuli.
c. Akinetic mutism is a hypomobile and mute syndrome wherein patients make few voluntary or responsive movements, although they blink, swallow, and may not appear distressed.
d. The locked-in state through involvement of the pons is a true coma where the patient has no means of producing speech or volitional limb movement

A

b. Stuporous patients can be transiently awakened by vigorous stimuli and avoid or withdraw from uncomfortable or aggravating stimuli.

Catatonia is hypomobile and mute syndrome that occurs usually as part of a major psychosis, typically schizophrenia or major depression. Patients with make few voluntary or responsive movements, although they blink, swallow, and may not appear distressed. There are nonetheless signs that the patient is responsive.

Akinetic mutism refers to a partially or fully awake state in which the patient is able to form impressions and think, as demonstrated by later recounting of events, but remains virtually immobile and mute. The condition results from damage in the regions of the medial thalamic nuclei or the frontal lobes (particularly lesions situated deeply or on the orbitofrontal surfaces) or from extreme hydrocephalus.

In a small proportion of patients in the vegetative state from posttraumatic cases, meaningful cerebral activation in response to verbal and other stimuli have been demonstrated by functional MRI.

The locked-in state describes an important type of pseudocoma in which an awake patient has no means of producing speech or volitional limb movement but retains voluntary vertical eye movements and lid elevation, thus allowing the patient to signal with a clear mind. The pupils are normally reactive.

259
Q

How long after a complete blood flow interruption will glucose supply to the brain last?

a. ~10 seconds
b. ~40 seconds
c. ~80 seconds
d. ~120 seconds

A

d. ~120 seconds

Cerebral neurons are dependent on cerebral blood flow (CBF) and the delivery of oxygen and glucose.

Brain stores of glucose are able to provide energy for ~2 min after blood flow is interrupted, and oxygen stores last 8–10 s after the cessation of blood flow.

Glucose = 2 mins/120m
Oxygen= 8-10sec

260
Q

Indications for lowering BP in px with stroke

A

BP > 220/120 OR
BP >185/110 and thrombolytic tx is anticipated

others
Malignant hpn
concomitantMI

261
Q

Most significant risk factor for stroke/TIA

A

Hypertension

Target <130/80
Preferred drugs: thiazide, diuretics, ACE-I

262
Q

Where do the eyes look in a hemispheral lesion of the brain?
a. Away
b. Toward
c. One eye away, one eye towards
d. Eye deviation is not specific to the area of lesion

A

b. Toward

263
Q

What type of breathing has a cyclic form that ends with brief apneic periods, and is seen in bi-hemispheral damage that commonly accompanies light coma?
a. Agonal gasps
b. Cheyne-Stokes respiration
c. Kussmaul breathing
d. Obstructive sleep apnea

A

b. Cheyne-Stokes respiration

Shallow, slow, but regular breathing suggests metabolic or drug depression.

Rapid, deep (Kussmaul) breathing usually implies metabolic acidosis but may also occur with pontomesencephalic lesions.

Agonal gasps are the result of lower brainstem (medullary) damage and are recognized as the terminal respiratory pattern of severe brain damage.

264
Q

Which of the following is NOT a sign of brain death?
a. Absent pupillary light reaction
b. Irreversible apnea
c. Loss of deep tendon reflexes
d. Unresponsiveness to all forms of stimulation

A

c. Loss of deep tendon reflexes

** should be loss of oculovestibular reflexes,

Occasionally other reflexes that originate from the spine may be present and should not preclude a diagnosis of brain death

265
Q

Which epilepsy syndrome is the most common syndrome associated with focal seizures with impairment of consciousness and is an example of an epilepsy syndrome with distinctive clinical, electroencephalographic, and pathologic features?
a. Autosomal dominant partial epilepsy with auditory features (ADPEAF)
b. Juvenile myoclonic epilepsy
c. Lennox-Gastaut syndrome
d. Mesial temporal lobe epilepsy (MTLE)

A

d. Mesial temporal lobe epilepsy (MTLE)

266
Q

A woman accidentally nicked her finger with a knife while she was cooking. Once her wound recovered she noticed she cannot feel hot or cold against the skin. Which fibers were likely severed or affected?
a. Autonomic nerves
b. Large diameter motor nerves
c. Large-diameter sensory fibers
d. Smaller diameter myelinated and unmyelinated fibers

A

d. Smaller diameter myelinated and unmyelinated fibers

Nerves can be subdivided into three major classes: large myelinated, small myelinated, and small unmyelinated.

Motor axons are usually large myelinated fibers that conduct rapidly (~50 m/s).

Sensory fibers may be any of the three types.
>Large-diameter sensory fibers conduct proprioception and vibratory sensation to the brain
>Smaller-diameter myelinated and unmyelinated fibers transmit pain and temperature sensation
>Autonomic nerves are also small in diameter.

DCML (capital letters –> large only)
TemP pAin –> (mixed small and large)

267
Q

A 56-year old diabetic complains of severe pain in the low back, hip, and thigh in one leg with weakness of that leg the past 7 weeks. Which type of neuropathy does he most likely have?
a. Distal sensorimotor polyneuropathy
b. Autonomic neuropathy
c. Diabetic radiculoplexus neuropathy
d. Diabetic mononeuropathies

A

c. Diabetic radiculoplexus neuropathy

Autonomic neuropathy is typically seen in combination with DSPN - abnormal sweating, dysfunctional thermoregulation, dry eyes and mouth, pupillary abnormalities, cardiac arrhythmias, postural hypotension, GI abnormalities (e.g., gastroparesis, postprandial bloating, chronic diarrhea, or constipation), and genitourinary dysfunction

Diabetic Radiculoplexus Neuropathy (Diabetic Amyotrophy or Bruns-Garland Syndrome) - presenting manifestation of DM in ~1/3 of patients; typically, patients present with severe pain in the low back, hip, and thigh in one leg; rarely, diabetic polyradiculoneuropathy begins in both legs at the same time. Atrophy and weakness of proximal and distal muscles in the affected leg become apparent within a few days or weeks. The neuropathy is often accompanied or heralded by severe weight loss. Weakness usually progresses over several weeks or months, but can continue to progress for 18 months or more. Subsequently, there is slow recovery but many are left with residual weakness, sensory loss, and pain

268
Q

If an electrodiagnosis shows axonal affectation, what is the more likely diagnosis?

a. Amyloidosis
b. Chronic inflammatory demyelinating polyradiculoneuropathy
c. Guillain-Barré syndrome
d. Vasculitis

A

d. Vasculitis

269
Q

Which statement is TRUE regarding therapy of intracranial malignancy?
a. Definitive treatment apply to brain tumors of any type.
b. Glucocorticoids are highly effective at reducing perilesional edema and improving neurologic function, often within hours of administration.
c. Prophylactic antiepileptic drugs are usually required and the agents of choice are those drugs that do not induce the hepatic microsomal enzyme system.
d. Prophylactic anticoagulants should be discouraged due to increased risk of hemorrhage into the tumor.

A

b. Glucocorticoids are highly effective at reducing perilesional edema and improving neurologic function, often within hours of administration.

Definitive treatment is based on the specific tumor type and includes surgery, radiotherapy, and chemotherapy. Symptomatic treatments apply to brain tumors of any type.

Most high-grade malignancies are accompanied by substantial surrounding edema, which contributes to neurologic disability and raised intracranial pressure.
Glucocorticoids are highly effective at reducing perilesional edema and improving neurologic function, often within hours of administration.
>Dexamethasone - glucocorticoid of choice because of its relatively low mineralocorticoid activity; initial doses are 8–16 mg/d.

Patients with brain tumors who present with seizures require antiepileptic drug therapy.
>Agents of choice are drugs that do not induce the hepatic microsomal enzyme system (levetiracetam, topiramate, lamotrigine, valproic acid, and lacosamide)

There is NO role for prophylactic antiepileptic drugs in patients who have not had a seizure.

Venous thromboembolic disease occurs in 20–30% of patients with high-grade gliomas or brain metastases. Prophylactic anticoagulants should be used during hospitalization and in nonambulatory patients.

Those who have had either a deep vein thrombosis or pulmonary embolus can receive therapeutic doses of anticoagulation safely and without increasing the risk for hemorrhage into the tumor.

Inferior vena cava filters are reserved for patients with absolute contraindications to anticoagulation such as recent craniotomy.

270
Q

Which is not a feature of critical illness polyneuropathy (CIP)?
a. It develops as a complication of sepsis and multiple organ failure.
b. Muscle stretch reflexes are present.
c. Serum creatine kinase (CK) is usually normal.
d. The usual presentation is inability to wean a patient from a ventilator.

A

b. Muscle stretch reflexes are present.

weakness developing in critically ill patients while in the ICU is usually caused by critical illness polyneuropathy (CIP) or critical illness myopathy (CIM) or, much less commonly, by prolonged neuromuscular blockade.

Both CIM and CIP develop as a complication of sepsis and multiple organ failure and usually present as an inability to wean a patient from a ventilator.

Muscle stretch reflexes are absent or reduced.

Serum creatine kinase (CK) is usually normal; an elevated serum CK would point to CIM as opposed to CIP.

NCS reveal absent or markedly reduced amplitudes of motor and sensory studies in CIP, whereas sensory studies are relatively preserved in CIM.

Needle EMG usually reveals profuse positive sharp waves and fibrillation potentials, and it is not unusual in patients with severe weakness to be unable to recruit motor unit action potentials.

271
Q

What is the most common type of malignant primary tumor?

a. Astrocytoma
b. Glioma
c. Medulloblastoma
d. Primary CNS lymphoma

A

b. Glioma

272
Q

A 38-year old male presented with ascending paralysis beginning as a feeling of rubbery legs. He had diarrhea 2 weeks prior. Which is NOT an accurate statement regarding his most probable condition?
a. Autonomic involvement such as wide fluctuations in blood pressure, postural hypotension, and cardiac dysrhythmias are common.
b. This manifests as a rapidly evolving areflexic motor paralysis with or without sensory disturbance.
c. Fever and constitutional symptoms are often present at the onset.
d. Pain in the neck, shoulder, back, or diffusely over the spine is seen in 50% of patients in the early stages.

A

c. Fever and constitutional symptoms are often present at the onset.

Guillain-Barre Syndrome
>Acute, frequently severe, and fulminant polyradiculoneuropathy; autoimmune in nature >Manifests as a rapidly evolving areflexic motor paralysis with or without sensory disturbance
>Usual pattern is an ascending paralysis that may be first noticed as rubbery legs.

> Weakness typically evolves over hours to a few days and is frequently accompanied by tingling dysesthesias in the extremities.
The legs are usually more affected than the arms, and facial diparesis is present in 50% of affected individuals.

> The lower cranial nerves are also frequently involved, causing bulbar weakness with difficulty handling secretions and maintaining an airway; the diagnosis in these patients may initially be mistaken for brainstem ischemia.

> Pain in the neck shoulder, back, or diffusely over the spine is also common in the early `stages of GBS, occurring in ~50% of patients.

> Most patients require hospitalization, and in different series, up to 30% require ventilatory assistance at some time during the illness.

273
Q

What is TRUE regarding the evaluation and diagnosis of myasthenia gravis?
a. The presence of anti-acetylcholine receptor antibodies is diagnostic but a negative test does not exclude the disease
b. If a patient has ptosis, application of a pack of ice over a ptotic eye often results in worsening of the ptosis
c. Repetitive nerve stimulation of weak muscles after anti-acetylcholinesterase medications are stopped 6–12 h prior may show evoked muscle action potentials that does not change by >10%.
d. Edrophonium chloride inhibits acetylcholinesterase that will cause immediate improvement and as a test should be done before subjecting the patient to nerve stimulation.

A

a. The presence of anti-acetylcholine receptor antibodies is diagnostic but a negative test does not exclude the disease

b. If a patient has ptosis, application of a pack of ice over a ptotic eye often results in worsening of the ptosis –> improves

c. Repetitive nerve stimulation of weak muscles after anti-acetylcholinesterase medications are stopped 6–12 h prior may show evoked muscle action potentials that does not change by >10%. –> Rapid reduction of >10% in the amplitude of the evoked responses in MG

d. Edrophonium chloride inhibits acetylcholinesterase that will cause immediate improvement and as a test should be done before subjecting the patient to nerve stimulation.–> done if other tests are neg

274
Q

A 60/M came into the ER for slurred speech and left-sided weakness one hour prior to consult. He was immediately brought to the ER for examination. On arrival, there was no note of any deficits. Physical examination was also unremarkable, with note of stable vitals and regular rhythm. Cranial CT scan was negative. Which of the following is true regarding the management of this patient?
a. Thrombolysis is indicated in this patient
b. Permissive hypertension is recommended for this patient to maximize the ischemic penumbra
c. Anticoagulation is recommended regardless of underlying risk factors
d. There is no evidence for the use of vitamin K antagonists over aspirin for secondary prevention of non-cardiogenic stroke

A

d. There is no evidence for the use of vitamin K antagonists over aspirin for secondary prevention of non-cardiogenic stroke

275
Q

A 60/M known diabetic was brought to the ER for seizures. He had a 3 day history of fever and headache which was not relieved with paracetamol. PE revealed stable vitals, fever at 39 deg C. He was awake but slightly confused. Neuro PE did not show focal deficits but was positive for nuchal rigidity. Lumbar tap was consistent with bacterial meningitis. What is the empiric regimen for this patient?

a. Ampicillin + cefotaxime
b. Ceftriaxone + vancomycin
c. Ampicillin + ceftriaxone + vancomycin
d. Ampicillin + meropenem + vancomycin

A

c. Ampicillin + ceftriaxone + vancomycin

c. Ampicillin + ceftriaxone + vancomycin

The patient’s presentation is consistent with bacterial meningitis, and he is at increased risk for Listeria monocytogenes due to his age (≥60 years) and diabetes mellitus.

275
Q

Which of the following electrolyte abnormalities is LEAST likely to cause delirium/encephalopathy?
a. Hypermagnesemia
b. Hyperglycemia
c. Hypocalcemia
d. Hyponatremia

A

a. Hypermagnesemia

275
Q

A 65-year old man with hypertension and dyslipidemia experienced an episode of dysarthria and left-sided weakness lasting for 30 minutes. He came to the ED fully recovered but his blood pressure was 165/100. Non-contrast cranial CT scan is normal and 12-L ECG shows normal sinus and rhythm. What is his ABCD 2 score and will you advise admission?
a. 4, no, admission not necessary
b. 4, yes, admission is necessary
c. 5, no, admission not necessary
d. 5, yes, admission is necessary

A

d. 5, yes, admission is necessary

65 - 1
30m -1
weakness -2
BP -1

> 3, admit

275
Q

Which of the following statements is TRUE about arteriovenous malformations?

a. Arteriovenous malformations are tufts of capillary sinusoids that form within the deep hemispheric white matter and brainstem with no normal intervening neural structures
b. Arteriovenous malformations consist of a tangle of abnormal vessels across the cortical surface or deep within the brain substance
c. Arteriovenous malformations are true capillary malformations that often form extensive vascular networks through an otherwise normal brain structure
d. Arteriovenous malformations are acquired connections usually from a dural artery to a dural sinus

A

b. Arteriovenous malformations consist of a tangle of abnormal vessels across the cortical surface or deep within the brain substance

275
Q

A 30-year-old female presented at the ED with a 2-day history of fever, headache, seizures, and confusion. She was suspected to have HSV encephalitis but her CSF PCR analysis was negative. What is the next course of action for this patient?

a. Discontinue acyclovir to prevent renal toxicity
b. Continue acyclovir and repeat the lumbar puncture
c. Repeat the PCR on the same sample
d. Send CSF to an outside laboratory to verify that the PCR is negative

A

b. Continue acyclovir and repeat the lumbar puncture

Herpes simplex virus (HSV) encephalitis is a medical emergency, and early treatment with acyclovir is critical to prevent severe neurological sequelae and death. A negative initial CSF PCR does not definitively exclude HSV encephalitis, particularly if the test was performed early in the disease course

275
Q

What is the standard criterion for treatment success in syphilitic meningitis

a. Completion of treatment with aqueous penicillin G in a dose of 3–4 million units intravenously every 4 h for 10–14 days
b. Complete resolution of clinical symptoms
c. Normal CSF cell count within 12 months and decrease in VDRL titer by two dilutions or revert to nonreactive within 2 years of completion of therapy
d. Persistence of CSF pleocytosis or an increase in the CSF VDRL titer by two or more dilutions after completion of therapy

A

c. Normal CSF cell count within 12 months and decrease in VDRL titer by two dilutions or revert to nonreactive within 2 years of completion of therapy

275
Q

Which of the following viruses causes tropical spastic paraparesis?

a. Cytomegalovirus
b. Epstein-Barr virus
c. Arbovirus
d. Human T-cell lymphotropic virus-1

A

d. Human T-cell lymphotropic virus-1

Tropical spastic paraparesis (TSP), also known as HTLV-1-associated myelopathy (HAM), is a progressive neurologic disorder caused by Human T-cell lymphotropic virus-1 (HTLV-1). It primarily affects the spinal cord, leading to chronic myelopathy and progressive spastic weakness of the lower extremities.

275
Q

Which of the following is the gold standard for diagnosis of small-fiber neuropathy?

a. Electromyography (EMG)
b. Nerve conduction studies(NCS)
c. Skin biopsy
d. Sural nerve biopsy

A

c. Skin biopsy

The gold standard for diagnosing small-fiber neuropathy (SFN) is a skin biopsy. SFN primarily affects small unmyelinated (C fibers) and thinly myelinated (Aδ fibers) nerves, which are not well assessed by conventional nerve conduction studies (NCS) or electromyography (EMG).

276
Q

Most common metastatic tumor associated with ICH

A

choriocarcinoma, malignant melanoma, renal cell carcinoma, bronchogenic carcinoma

277
Q

Prodromal symptom of enlarging aneurysm in the junction of Pcom and ICA

A

CN 3 palsy with pupillary dilatation
Loss of ipsilateral reflex
focal pain above or behind the eye

278
Q

Prodromal symptom of enlarging aneurysm in the cavernous sinus

A

CN 6

279
Q

Leading cause of morbidity and mortality following aneurysmal SAH

A

Delayed cerebral ischemia (DCI)

Vasospasm is the narrowing of the arteries at the base of the brain following SAH. This may cause symptomatic ischemia and infarction in ~30% of patients and is the major cause of delayed morbidity and death.

Signs of DCI appear 4–14 days after the hemorrhage, most often at 7 days.

Nimodipine prevents ischemic injury

280
Q

What electrolyte abnormality should you watch out for in the 1st two weeks of SAH

A

hyponatremia due to CSW

Avoid free water restriction since will inc risk of stroke
Supplemental oral salt with normal saline will mitigate hyponatremia BUT often requires hypertonic saline

281
Q

Most effective COMT inhibitor

A

Tolcapone

Fatal hepatotoxicity thus requires periodic LFT monitoring

282
Q

Treatment for Frontotemporal dementia

A

Symptomatic
No tx to slow progression or improve ssx

283
Q

Hot cross buns sign is seen in what type of dementia

A

MSA

284
Q

Definition of chronic migraine

A

Lasts for >= 8 days, >=15 total days per month

285
Q

When should you consider preventive tx for migraine

A

> =4 attacks per month

286
Q

Definitive diagnostic test for for leptomeningeal mets

A

CSF cytology showing tumor cells

positive in only 50% on 1st LP
Most useful in hematologic malignancies

Also definitive: (+) tumor markers or markers of clonal proliferation (flow cytometry)

287
Q

Instances wherein meningeal sign may be false negative or positive

A

May be reduced or absent in elderly, very young, immunocompromised

False positive: cervical spine dse (older px)

288
Q

When should you repeat LP in a px with pneumococcal meningitis

A

Patients with S. pneumoniae meningitis should have a repeat LP performed 24–36 h after the initiation of antimicrobial therapy to document sterilization of the CSF.

Failure to sterilize the CSF after 24–36 h of antibiotic therapy should be considered presumptive evidence of antibiotic resistance.

Patients with penicillin- and cephalosporinresistant strains of S. pneumoniae who do not respond to intravenous vancomycin alone may benefit from the addition of intraventricular vancomycin.

The intraventricular route of administration is preferred over the intrathecal route because adequate concentrations of vancomycin in the cerebral ventricles are not always achieved with intrathecal administration.

289
Q

Gold std diagnostic test for TB meningitis

A

CSF culture

if viral: PCR

Additional note: Dexamethasone recommended in HIV neg px

290
Q

Treatment for cryptococcal meningitis

A

NON HIV, NON transplant
Induction
Amp B + flucytositosine x 4 weeks after 2 weeks negative CSF culture; extend to 6w if with neuro complications

Consolidation
Fluconazole x 8 weeks
————————————–

Organ transplant px
Induction
Lip Amp B + Flucyosine at least 2 weeks or until CSF is sterile

Consolidation
Fluconazole 8-10w
————————————-

HIV +
Amp B + lipo formulation + Flucytosine x 2 week followed by Fluco x 8 weeks
May require indefinite maintenance tx with Fluconazole
————————————-

Most common complication: hydrocephalus

291
Q

Most common cause of chronic meningitis in immunoSUPRESSED

A

2 is Cryptococcus

TB

292
Q

CSF RBCs > 500/ uL in NON traumatic tap may be seen in what viral cause of encephalitis

A

HSV

293
Q

Drugs associated with progressive multifocal leukoencephalopathy

A

Rituximab, Efalizumab

May have peripheral ring enhancement on MRI

294
Q

Virus associated with progressive multifocal leukoencephalopathy

A

JC virus
HIV—> classically non enhancing

295
Q

Most common etiologic organism of brain abscess

A

Streptococcus particularly viridans

if frontal abscess : Strep milleri

296
Q

Most common location of brain abscess

A

Temporal

297
Q

T/F
Lumbar puncture should be ordered in all px with brain abscess

A

False

No value in empyema and brain abscess
Also inc risk of herniation

298
Q

Treatment for brain abscess if non surgical

A

Community acquired –> 3rd gen + Metronidazole

Penetrating head trauma/ recent neurosurgical procedure –> cover for Pseudomonas (CAZ/MEM) + S. aureus (VAN)

Tx duration: Min 6- 8 weeks

Prophylactic anticonvulsant: min 3 months (12wks). Repeat EEG. If abnormal, continue; if normal may slowly withdraw

299
Q

How do you monitor px with brain abscess?

A

Serial MRI/CT scan monthly or 2x a month to document resolution

300
Q

Most common type of GBS

A

AIDP

Additional Note: Fever and constitutional ssx not seen at onset of GBS

GBS only has mild sensory deficits except for DTR and proprioception

301
Q

Triad of Miller Fisher variant of GBS

A

Ophthalmoplegia
Areflexia
Ataxia

90% have anti GQ1b antibodies

302
Q

Electrodiagnostic features of GBS

A

AIDP: prolonged F wave latencies (earliest) , prolonged distal latencies, reduced CMAP

AMAN and AMSAN: reduced amplitude of CMAP WITHOUT conduction slowing or prolongation of distal latencies

303
Q

Treatment for GBS

A

IVIG = plasmapheresis

IVIG = 5 daily infusions for total of 2g/kg BW
Plasmapheresis= 40-50 mk/kg plasma exchange daily for 4-5 days

No role for glucocorticoids

304
Q

Positive finding in repetitive nerve stimulation in MG

A

Rapid reduction in amplitude of the evoked response by > 10-15%

Acetylcholinesterases should be witheld 6-24 hrs prior to testing

305
Q

Positive finding in Edrophonium test in MG

A

Improvement after administering edrophonium (1st give 2 mg, if no improvement, give 8 mg)

306
Q

Antibodies positive in MG

A

Anti ACh R –> positive in 85-90% of px; virtually diagnostic of MG

Anti MuSK –> 40% of ACh R neg px with generalized MG

others: Anti LRP4 (small proportion), anti-agrin

307
Q

Most common malignancy associated with LEMS

A

SCLCA

LEMS would have incremental response to repetitive nerve stimulatio

308
Q

What medication should be avoided in px with MG who is on azathioprine

A

Allopurinol because the combination may result in myelosupression

309
Q

A 19-year-old female was brought to the clinic after she was observed to be staring off into space, followed by chewing and picking movements of the hands. The patient was confused after the episode and could not recall doing those movements. She has no comorbidities. Her mother was also diagnosed with seizure. How would you classify this seizure?
a. Atonic seizure
b. Absence seizure
c. Myoclonic seizure
d. Focal seizure with impaired awareness

A

d. Focal seizure with impaired awareness

The patient’s symptoms, including staring off into space, automatisms (chewing and picking movements), postictal confusion, and amnesia, are characteristic of a focal seizure with impaired awareness. This type of seizure originates in a specific region of the brain (commonly the temporal lobe) and affects consciousness.

310
Q

Which of the following features is more suggestive of a generalized tonic clonic seizure rather than a syncope?
a. Usually no immediate precipitating factors
b. Frequently presents with premonitory symptoms
c. Duration of tonic-clonic movements usually lasts 10 to 15 seconds
d. Transition to unconsciousness is gradual over seconds except in cases of cardiac arrhythmia

A

a. Usually no immediate precipitating factors

311
Q

The family of a 70-year-old male being treated for Parkinson’s Disease for the last 10 years has now noticed that the patient became short tempered in the last few weeks. He is easily agitated by the crying and shouting of the toddlers at home and admits to having visual hallucinations seeing \gures resembling his deceased siblings. What would you prescribe to help improve his symptoms?
a. Fluoxetine
b. Risperidone
c. Clonazepam
d. Primavanserin

A

d. Primavanserin

Pimavanserin is frequently helpful to treat the psychosis and does not worsen parkinsonism Selective inverse agonist of the serotonin 5-HT2A receptor that does not block dopamine receptors

Low-dose clozapine is also effective for treating hallucinations and delusions, but requires frequent blood draws due to the risk of agranulocytosis

CLOZAPINE not clonazepam

312
Q

A 66-year-old woman diagnosed Parkinson’s disease has been receiving treatment for more than 5 years and on regular follow-up every 3-6 months. On her last follow-up, about 3 months ago, her attending physician prescribed a new medication for her condition. 2 weeks prior to consult, patient had episodes of disorientation, visual hallucinations and shortened temper. She has also gone out with friends on 5 separate occasions in the last 2 weeks for shopping sprees and on one instance went to a casino and lost 100,000 pesos while gambling. What medication could possibly cause such behavioral changes?
a. Levodopa-Carbidopa
b. Selegeline
c. Ropinorole
d. Entacapone

A

c. Ropinorole

The patient’s symptoms of disorientation, visual hallucinations, short temper, and impulsive behaviors (e.g., gambling and shopping sprees) are suggestive of impulse control disorders (ICDs) and dopaminergic psychosis. These behaviors are strongly associated with dopamine agonists, such as ropinirole.

313
Q

A 48-year-old male was brought to emergency room due to intermittent headache. Two weeks prior to his admission, he had a vehicular accident and suffered multiple contusion/hematoma on the face. He was observed by his family to be aphasic and had right-sided weakness. He also had several episodes of vomiting and noted to have increased sleeping time. Patient also had 2 episodes of generalized seizures. Cranial CT scan with contrast showed 1.0cm x 1.5cm rim-enhancing lesion on the left frontal lobe with prominent vasogenic edema. Cranial MRI with contrast showed increased signal on diffusion-weighted imaging (DWI) and low signal on apparent diffusion coepcient images. The appropriate medical management for this patient will be:
a. Ceftriaxone + Vancomycin + Mannitol + Dexamethasone
b. Ceftazidime + Vancomycin + Anti-convulsant + Mannitol
c. Ceftazidime + Metronidazole + Mannitol + Dexamethasone
d. Meropenem + Vancomycin + Anti-convulsant + Dexamethasone

A

d. Meropenem + Vancomycin + Anti-convulsant + Dexamethasone

If brain abscess with prior head trauma, may give
CZA / MEM + VAN

Steroids not routine unless with edema and inc ICP which are present in this case

314
Q

A 35-year-old male with history of multiple same-sex partners presented to the emergency room with behavioral changes. He was noted to have 3 episodes of focal seizures and on neurologic exam, he was noted to have leftsided weakness. Cranial MRI with contrast done showed multiple, non-enhancing white matter on both parietooccipital lobes. This patient most likely has: a. CNS lymphoma
b. Toxoplasmosis
c. Herpes encephalitis
d. Progressive Multifocal Leukoencephalopathy

A

d. Progressive Multifocal Leukoencephalopathy

90% of HIV PML - non ehnancing white matter

315
Q

A 58-year-old female came in due to seizures. On history, patient has no other symptoms aside from recurring headaches of unknown duration. Upon physical examination, patient was conscious and coherent, and no neurological dedcits were noted. Contrast-enhanced MRI was requested which showed a large cystic mass with central necrosis and surrounding edema. The treatment of choice for this patient would be:
a. Hypofractionated RT regimen over 3 weeks with temozolomide, followed by 6–12 months of adjuvant bevacizumab
b. Maximal surgical resection followed by partial-deld external-beam RT with concomitant temozolomide, followed by 6–12 months of adjuvant bevacizumab
c. Maximal surgical resection followed by partial-deld external-beam RT with concomitant temozolomide, followed by 6–12 months of adjuvant temozolomide
d. Anti-epileptic medications and observation of the mass with serial MR

A

c. Maximal surgical resection followed by partial-deld external-beam RT with concomitant temozolomide, followed by 6–12 months of adjuvant temozolomide

Glioblastomas are the most common malignant primary brain tumor. Patients usually present in the sixth and seventh decades of life with headache, seizures, or focal neurologic deficits. The tumors appear as ring-enhancing masses with central necrosis and surrounding edema.

Treatment involves maximal surgical resection followed by partial-field external-beam RT (6000 cGy in thirty 200-cGy fractions) with concomitant temozolomide, followed by 6 months of adjuvant temozolomide. With this regimen, median survival is increased to 14.6–18 months compared to only 12 months with RT alone, and 5-year survival is ~10%.

316
Q

Which of the available prophylactic migraine treatments is the usual culprit to cause glaucoma?
a. Propranolol
b. Metoprolol
c. Topiramate
d. Pizotifen

A

c. Topiramate

can also cause renal stones

317
Q

A 25-year-old male experienced sudden-onset of quick head drop with associated brief impairment of consciousness but without postictal confusion. EEG was done which showed brief, generalized spike-and-wave discharges followed immediately by diffuse slow waves. Which among these medications is considered as first-line of treatment for his condition?
a. Phenytoin
b. Clonazepam
c. Carbamazepine
d. Valproic Acid

A

d. Valproic Acid

The patient’s symptoms (sudden head drop, brief impairment of consciousness without postictal confusion) and EEG findings (generalized spike-and-wave discharges followed by slow waves) are consistent with myoclonic seizures or atonic seizures. These seizures are often part of generalized epilepsy syndromes, such as juvenile myoclonic epilepsy (JME) or Lennox-Gastaut syndrome (LGS).

Valproic acid is a first-line treatment for generalized seizures, including:
>Myoclonic seizures.
>Atonic seizures.
>Absence seizures.
>Generalized tonic-clonic seizures

318
Q

A 21-year-old female experienced blank stare lasting for 2 minutes with associated rapid blinking of the eyelids, chewing movements, lip smacking and clonic movements of the hands. EEG was done which showed a generalized, slow spike-and-slow-wave pattern with a frequency of ≤2.5 per second. Which among these medications is considered as first-line of treatment for her condition?
a. Phenytoin
b. Topiramate
c. Phenobarbital
d. Levetiracetam

A

b. Topiramate

Atypical absence –> Valproic, Lamotrigine, Topiramate
Typical –> Valproic , Lamotrigine, Ethosuximide

319
Q

Who among these patients should not be given Intravenous Recombinant Tissue Plasminogen Activator?
a. 56/F, large Middle Cerebral Artery infarct, BP=160/100mmHg, s/p EGD and treatment for bleeding duodenal ulcer 2 months ago
b. 58/M, Left Posterior Circulation infarct, BP=200/100 on admission, present BP is 170/100mmHg on Nicardipine drip
c.60/F, Right Anterior Cerebral Artery infarct, BP=180/100mmHg, s/p Abdominopelvic resection for rectal cancer 3 months ago
d.62/M, Pontine infarct, BP=160/110mmHg, on his 3rd hospital day, initially admitted for NSTEMI

A

d.62/M, Pontine infarct, BP=160/110mmHg, on his 3rd hospital day, initially admitted for NSTEMI

Since with recent MI

Contraindications
BP > 185/110
GI bleed within 3 weeks
Major GI surgery within 2 weeks

320
Q

A 64-year-old male, known Diabetic and Hypertensive but poorly compliant to medications was brought to the ER due to decreased sensorium. 30 minutes prior to consult, patient had sudden-onset of left sided weakness with associated slurring of speech. Subsequently, patient was noted to be drowsy. On examination, patient is stuporous and his eyes deviate away from the side of the hemiparesis. The affected limbs are flaccid. Where is the most likely localization of the lesion?
a. Pons
b. Putamen
c. Thalamus
d. Cerebellum

A

b. Putamen

Key Features of Hypertensive Putaminal Hemorrhage:
Hemiparesis: Contralateral to the lesion.
Gaze Deviation: Eyes deviate away from the side of weakness.
Altered Consciousness: Common with larger hemorrhages.
Flaccidity: Early phase due to disruption of motor pathways.

320
Q

A 45-year-old female sought consult due to intermittent episodes of headache characterized as throbbing, boring and stabbing, retroorbital in location, excruciating in intensity and associated with lacrimation and nasal congestion. She claimed that she would have 5 to 6 episodes of attacks per day and each attack would lasts 5 to 15 minutes. What is the treatment of choice for her condition?

a.Lithium
b.Topiramate
c.Lamotrigine
d.Indomethacin

A

d.Indomethacin

Paroxysmal hemicrania
THROBBING , >5x per day, 2-30mins

No mention of periodicity hence not cluster.
A core feature of cluster headache is periodicity. At least one of the daily attacks of pain recurs at about the same hour each day for the duration of a cluster bout. The typical cluster headache patient has daily bouts of one to two attacks of relatively short-duration unilateral pain for 8–10 weeks a year; this is usually followed by a pain-free interval that averages a little less than 1 year

321
Q

A 48-year-old male sought consult due to headache characterized as excruciating in intensity, nonfluctuating, and explosive in quality. The pain is retroorbital in location and daily attacks of pain recurs at about the same hour each day, about one to two attacks of relatively short-duration for 8–10 weeks a year which is followed by a pain-free interval of about 6 to 8 months. He tends to move about during attacks and rubbing his head would somehow provide relief. The headache was associated with lacrimation, aural fullness and rhinorrhea. What is his most likely diagnosis?
a. Cluster Headache
b. Hemicrania Continua
c. Paroxysmal Hemicrania
d. Short-lasting Unilateral Neuralgiform Headache attacks with Cranial Autonomic features

A

a. Cluster Headache

A core feature of cluster headache is periodicity. At least one of the daily attacks of pain recurs at about the same hour each day for the duration of a cluster bout. The typical cluster headache patient has daily bouts of one to two attacks of relatively short-duration unilateral pain for 8–10 weeks a year; this is usually followed by a pain-free interval that averages a little less than 1 year

322
Q

A 66-year-old male presented with rest tremor, rigidity and bradykinesia with postural instability. He was also observed to have mood swings and difficulty remembering things. On further examination, patient has freezing of gait, speech difficulty and swallowing impairment. You were asked to give a medication that reduces dyskinesia without worsening his present clinical features. What will you prescribe him?
a. Selegiline
b. Zonisamide
c. Amantadine
d. Benztropine

A

c. Amantadine

The only oral drug for dyskinesia that will not worsen parkinsonism

323
Q

A 68-year-old female presented with dementia, ataxic gait and urinary incontinence. Neuroimaging was done which revealed enlarged lateral ventricles with little or no cortical atrophy and the sylvian fissures appear propped open. What is her most likely diagnosis?
a. Huntington’s Disease
b. Korsakoff’s syndrome
c. Intracranial Hypotension
d. Normal-pressure hydrocephalus

A

d. Normal-pressure hydrocephalus

Wacky - Dementia
Wobbly- Ataxic
Wet - urinary incontinence

Triad of NPH - WWW

324
Q

A 62-year-old male, known alcoholic, presented with inability to recall new information despite normal immediate memory, attention span, and level of consciousness. He was also observed to be easily confused and had difficulty storing information for more than a few minutes. Patient is conversant, engaging, and able to perform simple tasks and follow immediate commands. What is his most likely vitamin deficiency?
a. B1
b. B6
c. B12
d. C

A

a. B1

Wernicke Korsakoff syndrome

325
Q

A 28-year-old immunocompetent female presented at the ER due to 4-day history of fever, headache, blurring of vision, progressive left hemiparesis, lethargy and seizures. Cranial MRI was done showing no hemorrhage and no infarct but with high signal intensity lesions in the orbitofrontal and temporal lobes. The next best step to do to determine the likely etiology is?
a. CSF Culture
b. CSF Serology
c. CSF PCR
d. CSF Microscopy

A

c. CSF PCR

Utility in both viral and bacterial especially since with involvement of temporal lobes (common in HSV)

Viral - low sensitivity viral culture

326
Q

A 25-year-old male known with HIV infection went to emergency room due to 1 month history of fever, headache, night sweats, lethargy and neck rigidity. He was referred to neurology service. CSF analysis was done showing eosinophilic pleocytosis, elevated protein concentration and decreased glucose concentration.
What is the most likely etiologic agent?
a.M. tuberculosis
b.C. immitis
c.L. monocytogenes
d.T. gondii

A

b.C. immitis

M. tuberculosis –> Mononuclear, may have PMN in early course of the dse
T. gondii –> Mononuclear

327
Q

A 21-year-old male with history of recurrent otitis media during his childhood years presented at the emergency room due to 5-day history of worsening headache, fever, aphasia and seizures. Which of the following has NO diagnostic value if you are considering a brain abscess?
a. CSF Analysis
b. Cranial MRI
c. Cranial CT Scan
d. Blood CS

A

a. CSF Analysis

CSF analysis no value in brain abscess. May inc risk of herniation

328
Q

EG, 40/M developed sudden onset slurring of speech and right sided weakness 40 minutes ago. He was then immediately brought to the ER. On the way to the hospital, his symptoms improved. At the ER, he was seen with a BP of 170/80 mmHg, HR 78. Neurologic examination showed no focal neurologic deficits. Plain cranial CT scan was normal. Assuming there are no contraindications, treatment includes:
a. Aspirin and Clopidogrel
b. Both IV thrombolysis and mechanical thrombectomy c. IV thrombolysis with rTPA only
d. Mechanical thrombectomy only

A

a. Aspirin and Clopidogrel

329
Q

A 60/F, obese, with family history of migraine developed bilateral throbbing headache after straining and stooping. What factor would favor more primary cough headache than primary exercise headache?
a. Age
b. Body built
c. Family history
d. Gender

A

a. Age

330
Q

What is the major genetic risk factor for Alzheimer’s Disease?
a. NFT allele of the E4 gene
b. ApoE allelle of the E4 gene
c. E4 allele of the ApoE gene
d. E4 allele of the NFT gene

A

c. E4 allele of the ApoE gene

331
Q

This type of dementia starts from one area of the brain, gradually spreading to other areas of the brain, such as from the Entorhinal area, to the hippocampal formation, limbic system, and to other basal temporal areas:
a. Vascular Dementia
b. Lewy body Dementia
c. Fronto Temporal Dementia
d. Alzheimer’s Disease Dementia

A

d. Alzheimer’s Disease Dementia

The pattern of binding is largely consistent with Braak neuropathologic staging of NFTs, with early retention in medial temporal regions, followed by spread into temporoparietal and cingu- late cortices, dorsolateral prefrontal regions, and, ultimately, primary sensory and motor areas

332
Q

A 65/M with a history of lung cancer presents with symptoms of leptomeningeal metastasis, including severe headaches, visual disturbances, and weakness in the lower extremities. Magnetic resonance imaging (MRI) of the brain and spine confirms the presence of leptomeningeal enhancement. The patient’s performance status is good. What is the most appropriate treatment approach for this patient?
a. Combined intrathecal chemotherapy and radiation therapy
b. Systemic chemotherapy
c. Stereotactic radiosurgery
d. Supportive care only

A

a. Combined intrathecal chemotherapy and radiation therapy

333
Q

Leptomeningeal metastases are more commonly associated with which type of primary cancer?
a. Breast cancer
b. Colorectal cancer
c. Pancreatic cancer
d. Prostate cancer

A

a. Breast cancer

334
Q

A 25/M was admitted for fever and seizures. MRI was done revealing increased signal intensity in the frontotemporal region of the brain. Lumbar puncture was done with the following initial results: WBC 8 (neutrophils 30%, lymphocytes 70%), RBC 0, gram stain: no organism seen, no fungal elements noted. What EEG findings do you expect to see in this case?
a. Generalized slowing of waves in the anterior lobes.
b. Periodic, stereotyped, sharp-and-slow complexes
repeating at regular intervals 2-3 seconds.
c. Theta waves with mild slowing of the background rhythm.
d. Theta and delta waves are slowing.

A

b. Periodic, stereotyped, sharp-and-slow complexes
repeating at regular intervals 2-3 seconds.

Distinctive EEG pattern consisting of periodic, stereotyped, sharp-and-slow complexes originating in one or both temporal lobes and repeating at regular intervals of 2–3 s. Typically noted between days 2 and 15 of the illness and are present in two-thirds of pathologically proven cases of HSV encephalitis.

Acyclovir is of benefit in the treatment of HSV and should be started empirically in patients with suspected viral encephalitis, especially if focal features are present, while awaiting viral diagnostic studies. Treatment should be discontinued in patients found not to have HSV encephalitis, with the possible exception of patients with severe encephalitis due to VZV or EBV. HSV, VZV, and EBV all encode an enzyme deoxypyrimidine (thymidine) kinase that phosphorylates acyclovir to produce acyclovir-5′-monophosphate

335
Q

Eve, a 20/F, 14 weeks AOG was brought to emergency room due to 3 days history of fever, headache and myalgia. On the day of consult, PE revealed GCS score of 9. According to the husband, they attended a party 7 days ago and they ate cold cuts and cheese. What is the most likely etiologic agent?
a. Listeria monocytogenes
b. Haemophilus influenzae type B
c. Staphylococcus aureus
d. Streptococcus pneumoniae

A

a. Listeria monocytogenes

Risk group: Pregnant women and their neonate, older adults and immunocompromised persons

Source: Dairy products are an important source, especially soft cheese made with raw mily or produced from pasteurized milk in unsanitary facilities; ice cream and raw milk also have cause outbreaks

336
Q

Patrick, a 50/M, came in the ER due to febrile episodes which started 1 week prior to consult. Along with the fever, he also reported myalgia, generalized weakness, anorexia, nausea, frontal headache, photophobia and bilateral cervical lymphadenopathies. He is a known Hypertensive and Liver cirrhosis from chronic hepatitis B. Work ups revealed normal Cranial CT scan results. To rule out meningitis, a lumbar tap was done and CSF studies showed lymphocytic pleocytosis with atypical lymphocytes, elevated protein, and normal glucose. Opening pressure was slightly elevated. What is the most likely etiologic agent of his meningitis?
a. Bacteria
b. EBV
c. Enterovirus
d. HSV

A

b. EBV

Atypical lymphocytes in the CSF or peripheral blood is suggestive of EBV infection

337
Q

A 40/F came in at the ER due to decrease in sensorium and seizure. History with the patient’s husband revealed that she had been having headache and fever for the past 2 weeks and was self-medicating only with Paracetamol. Until few hours prior to consult, she had seizure prompting ER consult. Vital signs BP 110/60, CR 103, RR 24, T 38, sats 94%, GCS 12. CT scan of the head revealed an abscess with ring-enhancing capsule. In what histopathologic stage is the patient’s current presentation?
a. Early cerebritis stage
b. Late cerebritis stage
c. Early capsule formation
d. Late capsule formation

A

d. Late capsule formation

Since patient has seizures and fever for 2 weeks (14 days) most likely px has capsule formation + gliosis which is seen in late capsule formation

The early cerebritis stage (days 1–3) is characterized by a perivascular infiltration of inflammatory cells, which surround a central core of coagulative necrosis. Marked edema surrounds the lesion at this stage.

In the late cerebritis stage (days 4–9), pus formation leads to enlargement of the necrotic center, which is surrounded at its border by an inflammatory infiltrate of macrophages and fibroblasts. A thin capsule of fibroblasts and reticular fibers gradually develops, and the surrounding area of cerebral edema becomes more distinct than in the previous stage.

The third stage, early capsule formation (days 10–13), is characterized by the formation of a capsule that is better developed on the cortical than on the ventricular side of the lesion. This stage correlates with the appearance of a ring-enhancing capsule on neuroimaging studies.

The final stage, late capsule formation (day 14 and beyond), is defined by a well-formed necrotic center surrounded by a dense collagenous capsule. The surrounding area of cerebral edema has regressed, but marked gliosis with large numbers of reactive astrocytes has developed outside the capsule. This gliotic process may contribute to the development of seizures as a sequela of brain abscess

338
Q

A 25/F presents with sudden onset of fever, severe headache, neck stiffness and altered mental status. The patient appears confused and lethargic. A lumbar puncture is performed, and the cerebrospinal fluid analysis reveals an elevated white blood cell count, elevated protein levels and normal glucose concentration. What is the most appropriate initial management for this patient?
a. Ceftazidime and Linezolid
b. Ceftriaxone and Dexamethasone
c. Piperacillin Tazobactam and Amikacin
d. Vancomycin and Ceftriaxone

A

d. Vancomycin and Ceftriaxone

339
Q

CL, a 26-year-old-male, was admitted for a 2-week history of headache with undocumented fever, progressing to generalized tonic-clonic seizures. He is a kidney allograft recipient and in maintained on steroids and tacrolimus. He was received lethargic, with left hemiparesis. MRI revealed ring-enhancing abscess, right hemisphere. Stereotactic aspiration was performed, and initial microbiologic tests showed +4 pus cells, no microorganisms seen; no fungal elements seen; MTb not detected on GeneXpert; weakly positive for acid-fast organisms. What is the most likely etiologic agent in this case?
a. Mycobacterium tuberculosis
b. Nocardia spp
c. Non-tuberculous mycobacteria
d. Toxoplasma gondii

A

b. Nocardia spp

340
Q

A 79/M is brought to your clinic because the son noticed his dad frequently talking to his mom who has passed away 8 years ago. The patient was also noted to be more depressed with poor attention span. He also had frequent falls especially in the front steps of his house. His head MRI shows atrophy of the amygdala. A polysomnogram shows rapid eye movement sleep behavior disorder with atonia. He most probably is suffering from:
A. Alzheimer’s Disease
B. Parkinson’s Disease
C. Vascular Dementia
D. Dementia with Lewy Bodies

A

D. Dementia with Lewy Bodies

REM and Visual problems = Lewy body

341
Q

72/F with chronic diabetes and hypertension with history of mild ischemic stroke 7 years ago but no residuals was noted to have frequent memory lapses and frequent falls causing her to withdraw from the outside world, preferring to stay at home and isolate herself from her family and friends. What are the basic routine tests that you should order?
A. CBC, chest xray, crea, EEG, head MRI
B. CBC, electrolytes, TSH, Vit B12, head CT scan
C. CBC, crea, RBS, lumbar tap, head MRI
D. CBC, ECG, chest xray, urinalysis, head CT scan

A

B. CBC, electrolytes, TSH, Vit B12, head CT scan

342
Q

67/M brought to your clinic by his wife due to changes in behavior. He was previously described as a soft-spoken, well-mannered man but the wife reports episodes where he would just suddenly pull down his pants and urinate in public while cursing at those who look his way. He has hypertension with previously good compliance to medications but lately he would turn violent when asked to take his BP meds. She also noted that he developed a voracious appetite and even witnessed him eating tissue paper. Which of the following is his most likely condition?
A. Alzheimer’s disease
B. Frontotemporal Dementia
C. Vascular Dementia
D. Dementia with Lewy Bodies

A

B. Frontotemporal Dementia

343
Q

Which of the following proteins or neurotransmitters decrease in patients with Alzheimer’s Disease?
A. Acetylcholine
B. Dopamine
C. Vitamin B12
D. Vasopressin

A

A. Acetylcholine

Biochemically, AD is associated with a decrease in the cortical levels of several proteins and neurotransmitters, especially acetylcholine, its synthetic enzyme choline acetyltransferase, and nicotinic cholinergic receptors

Dopamine is decreased in PD

344
Q

A Medicine resident on duty sought consult at the ER because of right sided temporal headache, described as throbbing and aggravated by moving her head down with associated photophobia which all started three days ago. There were no other associated signs and symptoms such as weakness nor dizziness. What is the best initial treatment for her?
A. NSAIDs
B. 5HT Receptor Agonist
C. CGRP Receptors Antagonist
D. Dopamine Receptor Antagonist

A

A. NSAIDs

345
Q

Which is commonly seen in the first phase of a migraine attack?
A. Yawning
B. Photophobia
C. Allodynia
D. Tinnitus

A

A. Yawning

Photophobia and Allodynia = headache phase (2nd phase)

The first phase of a migraine attack for most patients is the premonitory (prodromal) phase consisting of some or all of the following: yawning, tiredness, cognitive dysfunction, mood change, neck discomfort, polyuria, and food cravings; this can last from a few hours to days.

Typically, the headache phase follows with its associated features, such as nausea, photophobia, and phonophobia as well as allodynia. When questioned, these typical migraine symptoms also emerge in the premonitory phase, and typical premonitory symptoms also continue into the headache phase.

As the headache lessens, many patients enter a postdrome, most commonly feeling tired/weary, having problems concentrating, and experiencing mild neck discomfort that can last for hours and sometimes up to a day

346
Q

A patient consults your clinic due to photophobia and nasal congestion. She also complains of moderate to severe vertigo with mild headache. Laboratory work up done were all unremarkable including head CT Scan. What type of primary headache does this patient most likely have?
A. Migraine with Aura
B. Migraine with Brainstem aura
C. Acephalgic Migraine
D. Visual Snow syndrome

A

C. Acephalgic Migraine

Patients with acephalgic migraine experience recurrent neurologic symptoms, often with nausea or vomiting, but with little or no headache. Vertigo can be prominent; it has been estimated that one-third of patients referred for vertigo or dizziness have a primary diagnosis of migraine.

347
Q

A 21-year-old male has been complaining of a dull headache and fever for 11 days. He self medicated with Co Amoxiclav and Paracetamol that offered minimal relief of the symptoms. At the emergency room he was seen drowsy, temperature 38.2C with weakness on the left arm and left leg but no sensory deficit. His informant recalled that he had a fall injury a month ago sustaining lacerated wound on his head that self-healed on continued dressing and application of povidone iodine. What is the recommended empiric antibiotic for the case?
A. Piperacillin Tazobactam + Vancomycin
B. Meropenem + Linezolid
C. Ceftazidime + Metronidazole + Vancomycin
D. Ampicillin + Vancomycin + Fluconazole

A

C. Ceftazidime + Metronidazole + Vancomycin

348
Q

A 51-year-old female presented with 3 days of fever and headache. At the emergency room, she was seen drowsy with BP 110/70 CR 82 RR 19 Temp 38.9C. Pertinent physical examination included positive nuchal rigidity but negative for Kernig’s and Brudzinski. Lumbar puncture was done and started empirically on Ceftriaxone. After 2 days, CSF culture revealed Neisseria meningitidis sensitive to all antibiotics. At this time, patient is afebrile, conscious and coherent with no nuchal rigidity. How will you continue her treatment?
A. Shift Ceftriaxone to Penicillin G to complete a total duration of 10 days
B. Continue Ceftriaxone to complete total duration of 7 days
C. Shift Ceftriaxone to Cefuroxime and complete a total duration of 14 days
D. Shift Ceftriaxone to Ceftazidime to complete total duration of 7 days

A

B. Continue Ceftriaxone to complete total duration of 7 days

Meningococcal - 7 days
Pneumococcal -14 days
G- and Listeria -21days

349
Q

A 54-year-old male came in at the Emergency Room with 30 minutes history of sudden onset left sided body weakness. On Physical Examination: BP 160/100 mmHg; HR 76 bpm; RR 20 cpm, Motor Testing: 5/5 on both right upper and lower extremities, 3/5 on both left upper and lower extremities; Sensory testing on the right is 100% while 50% on the left. What imaging modality will help in the diagnosis?
A. Cerebral Angiography
B. Cranial CT scan
C. Cranial MRI with DWI
D. Cranial Ultrasonography

A

B. Cranial CT scan

350
Q

A 50-year-old Female came in at Emergency Room with diplopia and hoarseness, other PE findings include: dizziness, numbness of the right side of the face and left side limbs, dysarthria, dysphagia, miosis on the right as well as ptosis on the right. What artery was involved?
A. Basilar Artery
B. Internal Carotid Artery
C. Middle Cerebral Artery
D. Posterior Inferior Cerebellar Artery

A

D. Posterior Inferior Cerebellar Artery

Wallenburg syndrome- PICA
Ipsilateral Horner and face
Contralateral limb

351
Q

A 49-year-old Female, hypertensive and diabetic came in at the Emergency Room with two hours history of sudden onset right sided body weakness. On Physical Examination: BP 180/90 mmHg; HR 70 bpm; RR 20 cpm, normal rate, regular rhythm, no murmurs; Motor testing: 3/5 on both right upper and lower extremities, 5/5 on both upper and lower left extremities; 50% sensory on the right and 100% on the left. Cranial MRI with DWI showed hyperintensity on the left corona radiata. What is the BEST treatment for this case?
A. Anti-thrombotic Therapy
B. Intravenous Thrombolysis
C. Neuroprotection
D. Rehabilitation

A

B. Intravenous Thrombolysis

Within golden perior. no contraindications to rtpa

352
Q

A 24-year-old female, known to have Seizure Disorder maintained on Levetiracetam came to with a complaint of increasing seizure episode during menstruation in spite of taking her medications. What is the BEST treatment for this case?
A. Add another anti-seizure medication
B. Change the anti-seizure medication
C. Lower the dose of the anti-seizure medication
D. Increase the dose of the anti-seizure medication

A

D. Increase the dose of the anti-seizure medication

The patient is experiencing catamenial epilepsy, where seizures are more frequent during menstruation due to hormonal fluctuations. In such cases, seizure control may worsen during specific phases of the menstrual cycle, even when the patient is compliant with their anti-seizure medication. Catamenial epilepsy is often managed by adjusting the dose of the anti-seizure medication during the menstrual cycle when seizures are more likely to occur. Increasing the dose during this period can help control the increased seizure activity.

353
Q

A 65-year-old Male known to have Parkinson’s Disease on Levodopa and Carbidopa for almost 3 years, the relatives noted that the patient had intermittent episodes of rigidity, tremors and involuntary movements. The BEST treatment is to add
A. Amantadine
B. Entacapone
C. Pramipexole
D. Selegiline

A

B. Entacapone

The patient is experiencing “wearing-off” or motor fluctuations, which are common in Parkinson’s disease after a few years of treatment with Levodopa and Carbidopa. This is characterized by periods of reduced efficacy of the medication, leading to the reappearance of symptoms like rigidity, tremors, and involuntary movements (dyskinesias).

Entacapone is a COMT (catechol-O-methyltransferase) inhibitor that helps to prolong the effect of Levodopa by inhibiting its breakdown, thereby improving motor symptoms and reducing “wearing-off” episodes.

A - is an option for managing dyskinesias but may not be the best initial approach for wearing-off fluctuations.

C - is a dopamine agonist, which could be considered, but adding a COMT inhibitor like Entacapone is usually the preferred first step when motor fluctuations occur.

D - is an MAO-B inhibitor that also helps to extend the effect of Levodopa, but Entacapone is generally more effective in managing wearing-off symptoms.

354
Q

A 35-year-old female presents to the emergency department with rapidly progressing weakness that started in her legs and is now affecting her arms. She reports a recent URTI a few weeks ago. On examination, she has symmetrical weakness, absent deep tendon reflexes, and mild sensory disturbances. Electromyography shows demyelination. What is the BEST management for this case?
A. Glucocorticoids
B. IVIg therapy
C. IVIg and PLEX therapy
D. Observe for progression of symptoms

A

B. IVIg therapy

Either IVIG or PLEX (Plasma exchange). Cannot be used in combination

IVIG usually first choice due to ease of administration and good safety record compared to PLEX

355
Q

A 60-year-old female consulted due to hyponasal voice that occurs in the afternoon. It is also associated with ptosis of the right eye, doubling of vision, and difficulty in swallowing. The symptoms are absent in the morning and occurs in afternoon and usually worse if the patient walks longer than her usual. On physical exam, VS were normal and pertinent findings include right lateral rectus paresis and ptosis. Application of a pack of ice over the right eye improves the ptosis. What is the BEST treatment for this case?
A. Glucocorticoids
B. IVIg
C. Plasmapheresis
D. Pyridostigmine

A

D. Pyridostigmine

Pyridostigmine is a cholinesterase inhibitor that increases the availability of acetylcholine at the neuromuscular junction, improving muscle strength. It is the first-line treatment for symptomatic management in Myasthenia Gravis.

356
Q

A 65-year-old Male diagnosed to have Prostate Cancer complained of back pain for the past month, usually worst when he lies down and relieved assuming a recumbent position. He also noticed progressive leg weakness and came for consult because of uncontrolled bowel movement. Plain MRI of the whole spine showed bone metastasis and epidural mass. What is the immediate treatment for this case?
A. Surgical Resection
B. Radiation therapy
C. Surgical Resection with subsequent Radiation therapy
D. Chemotherapy with Radiation therapy

A

C. Surgical Resection with subsequent Radiation therapy

If with EPIDURAL mass –> surgery then RT

RT remains mainstay of tx

357
Q

What vascular territory is involved in a patient who presents with sudden onset of right-sided sensory disturbance, mild right-sided arm weakness, prominent right-sided leg weakness, and mild dysarthria?
a. Inferior division of the left middle cerebral artery
b. Left anterior cerebral artery
c. Left posterior cerebral artery
d. Superior division of the left middle cerebral artery

A

b. Left anterior cerebral artery

Leg > Arm weakness = ACA

358
Q

A patient presented with sudden onset of hemiplegia, hemianesthesia, global aphasia, and preferential gaze to the left. Plain cranial CT scan revealed an infarct. After 2 days, the patient became comatose with a dilated and non-reactive left pupil. What is this syndrome?
a. Artery of Percheron infarction
b. Hyperacute middle cerebral artery infarction
c. Malignant middle cerebral artery infarction
d. Weber syndrome

A

c. Malignant middle cerebral artery infarction

The patient’s presentation of sudden hemiplegia, hemianesthesia, global aphasia, and gaze preference to the left is consistent with a large middle cerebral artery (MCA) infarction affecting the dominant hemisphere. The progression to coma and a dilated, non-reactive left pupil 2 days later suggests malignant MCA infarction with herniation due to massive cerebral edema.

359
Q

32/F patient came in due to a 4-day history of sudden severe headache. Subarachnoid hemorrhage (SAH) is considered but the plain cranial CT scan did not show blood in the subarachnoid space. What is the next step to diagnose SAH?
a. Four-vessel angiography
b. Lumbar puncture
c. Repeat the CT scan after 12 hours
d. Transcranial Doppler scan

A

b. Lumbar puncture

360
Q

What is the target international normalized ratio (INR) for a patient with bileaflet mechanical mitral valve who is receiving vitamin K antagonist (VKA) therapy to significantly reduce the risk for cardioembolic stroke?

a.1.5-2.5
b.2-3
c.2.5-3.5
d.3-4

A

c.2.5-3.5

For patients with a bileaflet mechanical mitral valve receiving vitamin K antagonist (VKA) therapy (e.g., warfarin), the target international normalized ratio (INR) is 2.5–3.5. This higher range is required due to the increased thromboembolic risk associated with prosthetic mitral valves compared to aortic valves.

361
Q

What is the reason why nimodipine is given to subarachnoid hemorrhage (SAH) patients?
a. Prevents hypotension
b. Prevents ischemic injury
c. Reduces headache severity
d. Reduces the risk of vasospasm

A

b. Prevents ischemic injury

DCI remains the leading cause of morbidity and mortality following aneurysmal SAH.

Treatment with the calcium channel antagonist nimodipine (60 mg PO every 4 h) improves outcome, perhaps by preventing ischemic injury rather than reducing the risk of vasospasm

362
Q

Which of the following radiologic features would suggest a cardioembolic etiology for stroke?
a. Generalized cerebral atrophy
b. Hemorrhagic transformation of the infarct
c. Hypodensity of an arterial territory on cranial CT scan d. Prominent perivascular spaces in the brain parenchyma

A

b. Hemorrhagic transformation of the infarct

. Cardioembolic strokes are often caused by emboli originating from the heart (e.g., atrial fibrillation, mechanical heart valves, or left ventricular thrombus) and tend to occlude large cerebral arteries.

The characteristics of these strokes make them more prone to hemorrhagic transformation due to: >Reperfusion injury after spontaneous clot dissolution. >Fragile ischemic brain tissue subjected to high-pressure arterial blood flow.

363
Q

What anti-seizure medication will have a reduced blood level when administered together with estrogen containing oral contraceptives?
a. Clonazepam
b. Gabapentin
c. Lamotrigine
d. Phenobarbital

A

c. Lamotrigine

Lamotrigine levels are significantly reduced when administered with estrogen-containing oral contraceptives. This interaction occurs because estrogen induces the synthesis of hepatic glucuronyltransferase enzymes, which enhance the metabolism of lamotrigine through glucuronidation. As a result, lamotrigine clearance increases, leading to lower plasma levels and reduced therapeutic efficacy.

364
Q

A patient presented with sudden and brief muscle contractions of the right lower extremity. What are the EEG findings that can be correlated with this type of seizure?
a. Bilaterally synchronous spike and slow wave discharges immediately prior to appearance of muscle artifacts.
b. Bursts of generalized, symmetric, 3-Hz spike and slow wave discharges provoked by hyperventilation.
c. Focal spike and slow waves seen maximally at the anterior temporal lobes.
d. Progressive increase in low-voltage fast activity followed by generalized high amplitude polyspike discharges.

A

a. Bilaterally synchronous spike and slow wave discharges immediately prior to appearance of muscle artifacts.

Myoclonic Seizure

Myoclonus is a sudden and brief muscle contraction that may involve one part of the body or the entire body. Pathologic myoclonus is most commonly seen in association with metabolic disorders, degenerative CNS diseases, or anoxic brain injury.

The EEG shows bilaterally synchronous spike-and-slow-wave discharges immediately prior to the movement and muscle artifact associated with the myoclonus.

Myoclonic seizures usually coexist with other forms of generalized seizures but are the predominant feature of juvenile myoclonic epilepsy (JME)

365
Q

Paroxysmal depolarizing shifts of the neuronal membrane lead to the opening of which ion channel?
a. Calcium
b. Gamma-amino butyric acid
c. Potassium
d. Sodium

A

d. Sodium

The bursting activity in individual neurons (the “paroxysmal depolarization shift”) is caused by a relatively long-lasting depolarization of the neuronal membrane due to influx of extracellular calcium (Ca2+), which leads to the opening of voltage-dependent sodium (Na+) channels, influx of Na+, and generation of repetitive action potentials

Influx of Calcium but it is the sodium channels which will open

366
Q

18/F without comorbidities presented with focal onset seizures accompanied by impairment of consciousness and post-ictal disorientation. Cranial magnetic resonance imaging showed sclerosis at the left hippocampus. What is the most likely diagnosis?
a. Atypical Absence Epilepsy Syndrome
b. Dravet Syndrome
c. Lennoux-Gestaut Syndrome
d. Mesial Temporal Lobe Epilepsy

A

d. Mesial Temporal Lobe Epilepsy

Mesial temporal lobe epilepsy (MTLE) is the most common syndrome associated with focal seizures with impairment of consciousness and is an example of an epilepsy syndrome with distinctive clinical, EEG, and pathologic features

High-resolution magnetic resonance imaging (MRI) can detect the characteristic hippocampal sclerosis that appears to be essential in the pathophysiology of MTLE for many patients

367
Q

27/F diagnosed with juvenile myoclonic epilepsy came into your clinic for pregnancy counseling. She is currently in her 6th week of pregnancy. She was maintained with levetiracetam 1 gram per day for the last 10 years with seizures occurring once or twice a year. She is concerned that her medications can have significant effects on her baby. What is the best treatment plan for her anti-seizure medication?
a. Decrease the dose of levetiracetam to 500 mg/day for 4 weeks then discontinue.
b. Discontinue levetiracetam and resume after her pregnancy at 6 weeks postpartum.
c. Maintain the dose of levetiracetam and add folic acid at 4 mg/day.
d. Shift levetiracetam to topiramate 50 mg/day

A

c. Maintain the dose of levetiracetam and add folic acid at 4 mg/day.

Because the potential harm of uncontrolled convulsive seizures on the mother and fetus is considered greater than the teratogenic effects of antiseizure drugs, it is currently recommended that pregnant women be maintained on effective drug therapy.

When possible, it seems prudent to have the patient on monotherapy at the lowest effective dose, especially during the first trimester.

For some women, however, the type and frequency of their seizures may allow for them to safely wean off antiseizure drugs prior to conception.

Patients should also take folate (1–4 mg/d), because the antifolate effects of anticonvulsants are thought to play a role in the development of neural tube defects, although the benefits of this treatment remain unproved in this setting.

367
Q

4/F diagnosed with focal epilepsy from left parietal lobe cavernoma has been experiencing episodes of fatigue and feeling of emptiness for the last 2 months. On further evaluation, she also had episodes of difficulty sleeping and has been unable to concentrate on her daily job as an accountant. She also gained weight and noticed that some of her clothes didn’t fit her anymore, which further aggravated her depressed mood. She plans to consult a psychiatrist soon. Which of the following medications may have significant interactions with her antiseizure medications?
a. Aripiprazole
b. Fluvoxamine
c. Imipramine
d. Venlafaxine

A

b. Fluvoxamine

Among the options, fluvoxamine is most likely to interact with ASMs because it is a potent inhibitor of CYP1A2, CYP2C19, and CYP3A4, enzymes involved in the metabolism of many ASMs

368
Q

Which of the following structures of the central nervous system is insensitive to pain?
a. Dural sinuses
b. Falx cerebri
c. Brain
d. Pial arteries

A

c. Brain

Relatively few cranial structures are pain producing; these include the scalp, meningeal arteries, dural sinuses, falx cerebri, and proximal segments of the large pial arteries.

The ventricular ependyma, choroid plexus, pial veins, and much of the brain parenchyma are not pain producing

369
Q

64/M with a prior history of CVD infarction of the right posterior cerebral artery with an MRS (Modified Rankin Scale) of 0 presented with first-onset focal clonic seizures of the left upper extremity. Levetiracetam was then started at a dose of 500mg/tablet twice a day. He then came back to your clinic after 2 weeks when his daughter noticed he became more irritable and had frequent labile moods but no new onset focal deficits. What would be the appropriate management option?
a. Perform a repeat cranial CT scan to rule out a new stroke
b. Shift to another anti-seizure medication
c. Increase levetiracetam to 500 mg/tablet 1 tablet 3x a day
d. Start carbamazepine 200 mg/tablet 1 tablet TID on top of current regimen

A

b. Shift to another anti-seizure medication

Mood changes is an adverse effect of Levetiracetam

370
Q

Typical EEG ndings seen in metabolic coma:
A. Predominant high-voltage slowing (δ or triphasic waves) in the frontal regions
B. Widespread fast (β) activity
C. Variable 8- to 12-Hz activity
D. Resembles the normal α rhythm of waking but not altered by environmental stimuli

A

Predominant high-voltage slowing (δ or triphasic waves) in the frontal regions is typical of metabolic coma, as from hepatic failure, and widespread fast (β) activity implicates sedative drugs (e.g., benzo- diazepines).

371
Q

A 43-year-old man presented with fever, repeated vomiting, unsteady gait, increased tonicity of his right upper limb and paucity of speech for three days. He was treated in the local hospital with antibiotics. He had hyponatraemia, which was rapidly corrected with hypertonic saline from 97 to 119 mmol/L. He was transferred to another hospital because of progressive reduction of consciousness, rapidly worsening rigidity, bradykinesia of all four limbs and worsening dysarthria. If the primary consideration is osmotic demyelination syndrome, which of the following statements is most consistent with this diagnosis?
A. MRI reveals a symmetric area of abnormal high signal intensity within the basis pontis
B. Pathology consists of demyelination with inflammation in the base of the pons, axons and nerve cells
C. MRI reveals abnormal enhancement of the mammillary bodies
D. The characteristic clinical triad is ophthalmoplegia, ataxia, and global confusion

A

A. MRI reveals a symmetric area of abnormal high signal intensity within the basis pontis

372
Q

First line treatment for focal seizure, EXCEPT
A. Carbamazepine
B. Levatiracetam
C. Topiramate
D. Lamotrigine

A

C. Topiramate

373
Q

Which of the following drugs is rst line in the management of status epilepticus?
A. Diazepam IV
B. Phenobarbital IV
C. Lorazepam IV infusion
D. Valproate IV

A

C. Lorazepam IV infusion

Lorazepam IV is considered the first-line treatment for status epilepticus due to its rapid onset of action and prolonged duration of effect compared to other benzodiazepines like diazepam.

Diazepam is also effective but is not preferred because it has a shorter duration of action and may require repeated doses or a continuous infusion to maintain seizure control.

374
Q

Withdrawal of anti-epileptic therapy in patients with completely controlled seizure can be done, EXCEPT
A. Complete medical control of seizures for 1 to 5 years
B. Single seizure type, either partial or generalized
C. Normal neurologic examination, excluding intelligence
D. Normal EEG

A

C. Normal neurologic examination, excluding intelligence

Discontinuation criteria:
(1) Complete medical control of seizures for 1 to 5 years (2) Single seizure type, either partial or generalized
(3) Normal neurologic examination, including intelligence (
4) Normal EEG

NOTE: all must be met for at least 2 years

374
Q

NOT correct about stroke syndromes :
A. Upper extremity weakness is more prominent in MCA occlusion
B. Agnosia, constructional apraxia and hemineglect is seen when the dominant hemisphere is involved
C. Hemiparesis is not a feature of vertebral artery occlusion
D. Memory decits can be seen for involvement of peripheral branches of PCA

A

B. Agnosia, constructional apraxia and hemineglect is seen when the dominant hemisphere is involved

MCA Dominant: aphasia
MCA Non-dominant: agnosia, constructional apraxia, hemineglect
ICA: sudden onset transient monocular blindness
ACA: LE>UE weakness, abulia, muteness, perseveration, disinhibition

375
Q

True about TB meningitis
A. It presents with acute onset fever, headache and meningismus
B. It develops acutely as a result of hematogenous spread of tubercle bacilli to the meninges
C. Development depends on the proximity of a caseous lesion to the subarachnoid space
D. Signs and symptoms primarily results directly from the invasion of the tubercle bacilli of brain structures

A

C. Development depends on the proximity of a caseous lesion to the subarachnoid space

Subacute meningitis/Chronic meningitis
Common causative organisms: M tuberculosis, C neoformans, H capsulatum, C immitis, T pallidum

SSx Days to weeks: unrelenting headache, sti-neck, low-grade fever, lethargy, night sweats, cranial nerve abnormalities

376
Q

NOT a feature of Alzheimer’s Dementia
A. End-stage disease may have myoclonic jerks, hyperactive tendon reflexes and generalized seizures
B. Begins with memory loss then language and visuospatial deficits later
C. Focal or generalized seizures may herald Alzhemier’s dementia even prior to dementia onset
D. Language impairment initially with fluency then comprehension then naming

A

D. Language impairment initially with fluency then comprehension then naming

Memory loss rst then progresses to language and visuospatial defects –> Language impairment–> naming comprehension –> fluency

377
Q

Which of the following drugs for Parkinsonism can directly stimulate dopaminergic receptors in the brain?
A. Pramipexole
B. Levodopa
C. Selegiline
D. Amantadine

A

A. Pramipexole

378
Q

True about GBS
A. The usual pattern is descending paralysis with weakness evolving over hours to a few days
B. Diagnosis is clinical and includes at least 1 week of progressive weakness of 2 or more limbs, areflexia and exclusion of other causes
C. About 2 weeks after the first motor symptoms, it is not known whether immunotherapy is still effective
D. Plasmapheresis is superior to IVIg as treatment for GBS

A

C. About 2 weeks after the first motor symptoms, it is not known whether immunotherapy is still effective

379
Q

Which of the following is NOT TRUE regarding multiple sclerosis?
A. It is an autoimmune CNS disease characterized by chronic inflammation, demyelination, gliosis and neuronal loss
B. Lesions develop at different times and in different CNS locations
C. Bladder dysfunction is generally not seen, and when present should direct attention to other differentials
D. MRI will show characteristic abnormalities in more than 95% of patients

A

C. Bladder dysfunction is generally not seen, and when present should direct attention to other differentials

Bladder dysfunction present in >90%

380
Q

Which vitamin is decient in Multiple Sclerosis?
A. Vitamin A
B. Vitamin D
C. Vitamin B6
D. Vitamin B12

A

B. Vitamin D

381
Q

What is the most common form of Multiple Sclerosis?
A. Relapsing or Bout MS
B. Secondary progressive MS
C. Primary progressive MS
D. PRMS

A

What is the most common form of Multiple Sclerosis? A. Relapsing or Bout MS

3 clinical types of MS:
1. Relapsing or Bout MS (RMS): 90% of MS cases
2. Secondary progressive MS (SPMS): always begins as RMS
3. Primary progressive MS (PPMS): ∼10% of cases

382
Q

Which of the following statements is true regarding the etiology and pathogenesis of bacterial meningitis?
A. Among adults the most common etiologic agent is N. meningitides
B. Patients who are alcoholic are predisposed to having pneumococcal meningitis
C. Diabetes mellitus is a predisposing risk for S. aureus meningitides
D. Splenectomy is a predisposing risk for coagulase-negative staphylococcal meningitis

A

B. Patients who are alcoholic are predisposed to having pneumococcal meningitis

Risk factors for pneumococcal meningitis
Most important: pneumococcal pneumonia

§ Others: coexisting acute/chronic pneumococcal sinusitis/otitis media, alcoholism, DM, splenectomy, hypogammaglobulinemia, complement deciency, head trauma with skull fracture and CSF rhinorrhea

383
Q

A patient with symmetric bilateral vestibular hypofunction will most likely present with which of the following?
A. Intermittent bilateral hearing loss lasting hours
B. Loss of balance in the dark without vertigo
C. Transient upbeating-torsional nystagmus
D. Unidirectional horizontal nystagmus

A

B. Loss of balance in the dark without vertigo

384
Q

A 30-year-old man presents with “generalized weakness” of 2 weeks duration. He has no known illnesses and takes no medication. There are no sensorial and cognitive changes. On physical examination, patient is alert and oriented to 3 spheres. There is decreased muscle strength on all extremities, with a positive Babinski reflex.
A baseline CBC and serum electrolytes are normal. What is the most appropriate next step in the evaluation of this patient?
A. Brain CT
B. Brain MRI
C. EMG and NCS
D. Spinal MRI

A

D. Spinal MRI

385
Q

A 45-year-old woman presents to the ER with a generalized seizure. She has a history of breast adenocarcinoma and underwent a modified radical mastectomy 2 years prior. She was advised adjuvant chemo- and radiation therapy then but was lost to follow up. She has no other known co-morbidities. A plain cranial CT shows no evidence of metastatic tumors. Leptomeningeal metastasis is considered. Which of the following is TRUE in the diagnosis of this condition?
A. Cerebral angiogram can be definitive when there is subarachnoid enhancement on imaging.
B. CSF cytologic examination is often positive for tumor cells in only 50% of patients on the first lumbar puncture.
C. Leptomeningeal metastasis is a clinical diagnosis, and patients who present with compatible symptoms should undergo palliative radiation therapy.
D. MRI is considered the gold standard in the evaluation of patients with suspected leptomeningeal metastases.

A

B. CSF cytologic examination is often positive for tumor cells in only 50% of patients on the first lumbar puncture.

386
Q

Aside from isoniazid, what other drug used in the treatment of tuberculosis can lead to peripheral neuropathy manifesting as numbness with loss of large-fiber modalities on examination?
A. Ethambutol
B. Pyrazinamide
C. Rifampicin
D. Streptomycin

A

A. Ethambutol

387
Q

A patient diagnosed with myasthenia gravis has been taking pyridostigmine 100 mg every 6 hours religiously. He now presents at the ER for increasing weakness, diarrhea, and salivation. What would you recommend for this patient?
A. Decrease the pyridostigmine daily dose.
B. Increase the pyridostigmine frequency to every four hours.
C. Refer to TCVS for emergency thymectomy.
D. Shift the patient to edrophonium.

A

B. Increase the pyridostigmine frequency to every four hours.

388
Q

A 66-year-old woman from a local home for the aged is brought to ER for increasing drowsiness. They report a history of a fall 1 week prior but did not seek consult at the time. Over the last few days, they note reports of headache that fluctuates in severity, inattentiveness and mild right-sided weaknesses. She has hypertension and is on once-daily losartan. On PE, BP 160/80 mmHg, HR 70/min and regular. She is drowsy, responds to name calling, disoriented, and inconsistently follows command. There is mild rightsided hemiparesis. What is the most likely diagnosis?
A. Chronic subdural hematoma
B. Posterior reversible leukoencephalopathy
C. Subarachnoid hemorrhage
D. Traumatic axonal injury

A

A. Chronic subdural hematoma

389
Q

Which of the following is most consistent with the case definition of major depressive disorder (MDD)?
A. Daily symptoms of sadness, indifference and apathy for at least 2 weeks
B. Manic and hypomanic episodes
C. Recurrent anxiety after exposure to traumatic events D. Somatic symptoms that are distressing and pre-occupying

A

A. Daily symptoms of sadness, indifference and apathy for at least 2 weeks

390
Q

Which of the following is TRUE when doing a neurologic examination on patients with suspected neurologic disease?
A. An extensor plantar response is indicative of a lower motor neuron lesion.
B. Biceps, patellar and Achilles reflexes should be checked in all patients.
C. Orientation is tested by asking the person to state his or her name, location, and time. Among the three, location is usually the first to be affected.
D. When doing cranial nerve (CN) examinations, the bare minimum includes evaluating CN II, III, IV, VI, VII and XI

A

B. Biceps, patellar and Achilles reflexes should be checked in all patients.

391
Q

A 38-year-old man presents with first-onset generalized seizure, lasting 2 minutes. He has no known history of epilepsy. Laboratory investigations including CBC, serum chemistries, toxicology screen, CSF analysis and brain MRI are normal. There is no other evidence of neurologic dysfunction. An EEG done shows a posteriorly situated 9-Hz alpha rhythm that attenuates with eye opening. Which of the following is the most likely diagnosis?
A. Degenerative disease
B. Idiopathic seizure
C. Occult malignancy
D. Subacute CNS infection

A

B. Idiopathic seizure

The clinical presentation of a first-onset generalized seizure in an otherwise healthy 38-year-old man, with normal laboratory investigations, imaging, and CSF findings, strongly points to an idiopathic seizure. The EEG findings showing a posteriorly situated 9-Hz alpha rhythm that attenuates with eye opening indicate a normal background rhythm, which supports the diagnosis of idiopathic seizure

392
Q

A patient with known brain tumor was started on an anti-epileptic drugs (AED) after presenting with multiple episodes of focal seizures. After surgery, how long should he be maintained on his AED?
A. May be discontinued a week after surgery, provided there are no neurologic deficits
B. May be discontinued after 6 months as long as he has been completely seizure-free
C. May be discontinued after 12 months as long as he has been completely seizure-free
D. Should be continued indefinitely because of the risk of recurrent seizure due to surrounding gliosis

A

C. May be discontinued after 12 months as long as he has been completely seizure-free

393
Q

A 75-year-old man presents to the ER with a sudden onset of oculomotor palsy with contralateral tremors and ataxia. Which of the following is the most likely affected intracranial vessel?
A. Anterior choroidal artery
B. Internal carotid artery
C. Middle cerebral artery
D. Posterior cerebral artery

A

D. Posterior cerebral artery

Claude syndrome

394
Q

Which of the following can be used as preventive treatment for migraine?
A. Clonidine
B. Fluoxetine
C. Nimodipine
D. Topiramate

A

D. Topiramate

395
Q

According to the UK Brain Bank Criteria, which of the following clinical features is most consistent with the diagnosis of Parkinson’s disease?
A. Asymmetric motor abnormalities, rest tremors, and a good response to levodopa
B. Bradykinesia with rigidity, fluctuating alertness, and visual hallucinations
C. Disinhibition, motor speech impairment, and focal atrophy of frontal cortex on MRI
D. Personality changes, symmetric axial rigidity, and slow, toppling gait

A

A. Asymmetric motor abnormalities, rest tremors, and a good response to levodopa

396
Q

Among these possible therapeutic options for Bell’s palsy, which of them can improve functional outcomes?
A. Acyclovir/ valacyclovir
B. Cosmetic surgery
C. Glucocorticoids
D. None of the above; manifestations typically resolve with only symptomatic measures

A

C. Glucocorticoids

397
Q

Which of the following clinical features of multiple sclerosis is seen in at least 25% of cases?
A. Ataxia
B. Pain
C. Visual loss
D. Sensory loss

A

D. Sensory loss

Sensory loss, optic neuritis and weakness

398
Q

A 49-year-old man is evaluated for persistent sensorimotor symptoms. 5 months ago, he developed tingling and mild bilateral pain in the thighs followed by mild weakness and hand numbness. Over the next 3 months, his lower extremity weakness progressively worsened, and his gait became unstable. His speech, swallowing, and vision were unaffected. His weakness has plateaued within the past 2 months without any improvement. Pertinent in the medical history is diabetes mellitus treated with metformin. Family history is noncontributory. On PE, vital signs are normal. Extraocular movements and muscle tone are normal; no fasciculations are present. Diffuse areflexia is noted, with moderate bilateral symmetric weakness in the distal upper extremities and proximal and distal lower extremities. Decreased sensation to pinprick and vibration is noted in both feet. Results of nerve conduction studies show diffuse and severe non-uniform slowing of motor nerve conduction velocities and the presence of conduction blocks.
Which of the following is the most appropriate next step in the management?
A. Do a nerve biopsy
B. Test for paraprotein
C. Treat as chronic inflammatory demyelinating polyneuropathy (CIDP)
D. Treat as Guillain-Barre syndrome

A

C. Treat as chronic inflammatory demyelinating polyneuropathy (CIDP)